Фиксирует менее точно ошибка

Светило науки — 1143 ответа — 0 раз оказано помощи

Відповідь:

В каком случае при образовании слова допущена ошибка?

1) САМАЯ ВЫСОЧАЙШАЯ причина

Пояснення:

Слова самый, наиболее, наименее нельзя использовать с прилагательными в простой форме превосходной степени, например, самая вкуснейшая, самая наилучшая…

Вместо этих словосочетаний нужно использовать простую или составную превосходную степень, не смешивая их: вместо самая высочайшая  можно сказать самая высокая или высочайшая.

samirka5696

СРОЧНО!!!!!!!ДАЮ 40 БАЛЛОВ В каком случае при образовании слова допущена ОШИБКА? Ответ запишите
цифрой. РЯДОМ НАПИШИТЕ ПРАВИЛЬНЫЙ ОТВЕТ
1) САМАЯ ВЫСОЧАЙШАЯ вершина
2) ОБРЕТАЮЩИЙ покой
3) фиксирует МЕНЕЕ ТОЧНО
4) НАСТИГНУВ добычу

lanastlana

Светило науки — 4507 ответов — 3300 раз оказано помощи

Ответ: ошибка в 1.

1) самая высочайшая вершина — ошибка: соединение простой и сложной превосходных степеней сравнения.

Верно:

самая высокая вершина — сложная превосходная степень,

высочайшая вершина — простая превосходная степень.

Ответы и задания для вариантов РЯ2180101, РЯ2180102 стартовой диагностической работы статград по русскому языку 8 класс 2021-2022 учебный год, официальная дата проведения работы: 23.09.2021 (23 сентября 2021 год).

Тренировочные варианты (РЯ2180101-РЯ2180102): скачать

Ссылка для скачивания всех ответов: скачать

На выполнение диагностической работы статград по русскому языку даётся 45 минут. Работа состоит из 19 заданий.

Варианты РЯ2180101 РЯ2180102 по русскому языку 8 класс статград

Задания с варианта РЯ2180101:

1)В каком слове при постановке ударения допущена ОШИБКА? Ответ запишите цифрой.

  • 1) пОнятый
  • 3) черпАть
  • 2) позвонИшь
  • 4) квартАл

2)Какое из выделенных слов образовано приставочным способом (причастие и деепричастие следует считать отдельными частями речи)? Ответ запишите цифрой.

  • 1) ЗАКУПОРЕННАЯ бутыль
  • 3) ПРИЧАЛИВ к берегу
  • 2) ПОДБЕЖАЛИ к нам
  • 4) ИЗРЕДКА кивал

4)В характеристике какого из выделенных слов допущена ОШИБКА? Ответ запишите цифрой.

  • 1) ТАМ (жили) − наречие со значением места.
  • 2) ИГРАЯ (на гитаре) − деепричастие совершенного вида.
  • 3) КРАДУЩИЙСЯ (тигр) − действительное причастие настоящего времени.
  • 4) (Оставил) ЛИШЬ (рубашку) − модальная частица со значением ограничения.

5)В каком случае при образовании слова допущена ОШИБКА? Ответ запишите цифрой.

  • 1) МЕНЕЕ ЯРЧЕ
  • 2) ОБУРЕВАЕМЫЙ гневом
  • 3) сохнет БОЛЕЕ ДОЛГО
  • 4) ПРИВЫКНУВ к неудобствам

6)Найдите предложение с грамматической ошибкой. Ответ запишите цифрой. 1) Работник музея рассказал туристам об одной из скульптур Родена, находящейся на реставрации. 2) Благодаря оперативным действиям администрации документы были оформлены в срок. 3) Снимая угловую комнату на втором этаже, спасением от зимних холодов для нас стала печка-буржуйка. 4) По приезде в Москву Иван Андреевич поселился у своего приятеля.

7)В каком ряду в обоих словах пропущена одна и та же буква? Ответ запишите цифрой.

  • 1) слыш..щий, колыш..щийся
  • 2) ненавид..вший, немысл..мый
  • 3) посе..нный, зате..вший
  • 4) замуч..ны, увелич..вший

8)В каком предложении в слове на месте пропуска пишется НН? Ответ запишите цифрой.

  • 1) Во дворе стоял запах масля..ой краски.
  • 2) Мясо было щедро приправле..о зелёным луком.
  • 3) Жаре..ые на сале кусочки курицы были ещё горячими.
  • 4) Кожа..ая куртка стоила дорого.

9)В каком ряду в обоих случаях НЕ со словом пишется слитно? Ответ запишите цифрой.

  • 1) (не)доброжелатель, (не)видя дороги
  • 2) (не)ряшливый вид, (не)отшлифованные камни
  • 3) (не)вольник чести, оценки (не)выставлены
  • 4) ответ (не)точен, вовсе (не)просто запомнить

10)В каком предложении выделенное слово пишется слитно? Ответ запишите цифрой. 1) Не знаю, на ЧТО(БЫ) здесь ещё можно было обратить внимание. 2) Учитесь смотреть (В)ГЛУБИНУ вопроса, видеть его сущность. 3) Иван, (НЕ)СМОТРЯ на хромоту, шёл наравне с остальными. 4) (ПО)ВАШЕМУ, это будет справедливым?

11)В каком ряду в обоих случаях пропущена одна и та же буква? Ответ запишите цифрой. 1) ш..коладный, деш..вый 2) (сдать) зач..т, облеч..н (властью) 3) мужич..нка, (крепкая) беч..вка 4) (мягкий) ш..лк , (с) кирпич..м

12)В каком предложении на месте пропуска пишется буква И? Ответ запишите цифрой. 1) У меня н.. минуты свободного времени. 2) Кто только н.. обращался к нему! 3) Все ветви были покрыты н.. почками или бутонами, а пышными соцветиями. 4) Нигде н.. мерцал огонёк.

13)В каком варианте ответа правильно указано и объяснено наличие или отсутствие запятых (знаки препинания не расставлены)? Ответ запишите цифрой. Встали пораньше (1) и (2) наскоро позавтракав (3) стали собираться (4) на запланированную накануне (5) рыбалку. 1) 1, 3 − выделяется деепричастный оборот 2) 2, 3 − выделяется деепричастный оборот 3) 1 − разделяются однородные члены предложения, 2, 3 – выделяется деепричастный оборот 4) 2, 3 − выделяется деепричастный оборот, 4, 5 − выделяется причастный оборот

16)В предложении 12 найдите слово, имеющее в тексте следующее значение: «разрушительная катастрофа». Напишите это слово.

17)Среди предложений 8–13 найдите все простые предложения. Напишите номера этих предложений через запятую.

18)Каким членом предложения является выделенный фрагмент? В 1986 году группа японских аквалангистов, ПРОВОДЯ ИССЛЕДОВАНИЯ У ОСТРОВА ЙОНАГУНИ, обнаружила под водой загадочные массивные объекты.

19)Среди предложений 4–7 найдите предложение(-я) с причастным(-ыми) оборотом(-ами). Напишите номер(-а) этого(-их) предложения(-ий).

Задания с варианта РЯ2180102:

1)В каком слове при постановке ударения допущена ОШИБКА? Ответ запишите цифрой. 1) начАвший 3) взЯта 2) добелА 4) (вы) прАвы

2)Какое из выделенных слов образовано приставочным способом (причастие и деепричастие следует считать отдельными частями речи)? Ответ запишите цифрой. 1) ПРИСТРОЕННАЯ терраса 3) ВЫГЛАДИВ бельё 2) НАГЛУХО забито 4) ПОДОБРАЛИ на улице

4)В характеристике какого из выделенных слов допущена ОШИБКА? Ответ запишите цифрой. 1) ТОГДА (решили) − наречие со значением времени. 2) ПОВЕРНУВ (направо) − деепричастие совершенного вида. 3) РАССЕРЖЕННЫЙ (человек) − действительное причастие прошедшего времени. 4) (Это был) ИМЕННО (он) − модальная частица с уточняющим значением.

5)В каком случае при образовании слова допущена ОШИБКА? Ответ запишите цифрой. 1) САМАЯ ВЫСОЧАЙШАЯ вершина 2) ОБРЕТАЮЩИЙ покой 3) фиксирует МЕНЕЕ ТОЧНО 4) НАСТИГНУВ добычу

5)Найдите предложение с грамматической ошибкой. Ответ запишите цифрой. 1) Поднявшаяся в небо журавлиная стая сделала несколько кругов над озером. 2) Поэты сравнивают храм Покрова на Нерли с парусом, уносящемуся вдаль по безбрежным волнам времени. 3) Песня звучала всё сильнее и сильнее, покрывая шум дождя. 4) По возвращении из эмиграции поэт с удвоенной силой взялся за работу

6)В каком ряду в обоих словах пропущена одна и та же буква? Ответ запишите цифрой. 1) реж..щий (инструмент), (тяжело) дыш..щий 2) множ..мое (число), невид..мый 3) нагруж..нный, встрет..вший 4) закле..нный, раста..вший

7)В каком предложении в слове на месте пропуска пишется НН? Ответ запишите цифрой. 1) Наблюдать за гордой поступью петуха было исти..ым удовольствием. 2) Наскоро сложенные страницы были перепута..ы. 3) В досках торчали старые кова..ые гвозди прошлого века. 4) Глиня..ые таблички были все испещрены какими-то знаками.

8)В каком ряду в обоих случаях НЕ со словом пишется слитно? Ответ запишите цифрой. 1) (не)достоверность, (не)взяв денег 2) (не)долго грустил, грядки (не)политы 3) (не)постоянный вкус, (не)пристёгнутый пассажир 4) цена (не)высока, вовсе (не)лёгкое дело.

9)В каком предложении выделенное слово пишется слитно? Ответ запишите цифрой. 1) Он высказал ТО(ЖЕ) предположение, что и я. 2) Никто не имел (В)ВИДУ ничего плохого. 3) Он попросил, (ЧТО)БЫ я остался. 4) Я к тебе отношусь (ПО)ДРУЖЕСКИ.

10)В каком ряду в обоих случаях пропущена одна и та же буква? Ответ запишите цифрой. 1) ш..фёр, печ..нка 2) медвеж..нок, освещ..нный (фонарями) 3) ш..рох, (со) свеч..й 4) (с) плащ..м, ж..рдочка

11)В каком предложении на месте пропуска пишется буква И? Ответ запишите цифрой. 1) Я не мог н.. простить эту невинную шалость. 2) И о чём только н.. мечтает девушка в шестнадцать лет! 3) Это н.. просто риск – это опасное дело. 4) Больше я н.. разу его не видел.

13)В каком варианте ответа правильно указано и объяснено наличие или отсутствие запятых (знаки препинания не расставлены)? Ответ запишите цифрой. Скифы были хорошо знакомы ещё античным народам (1) и (2) населяя некогда южные территории современной России (3) оставили после себя (4) пользующееся ныне широкой известностью (5) культурное наследие.

16)В предложениях 3–4 найдите слово, имеющее в тексте следующее значение: «научное предположение, выдвигаемое для объяснения каких-нибудь явлений». Напишите это слово.

17)Среди предложений 7–12 найдите все простые предложения. Напишите
номера этих предложений через запятую. 

18)Каким членом предложения является выделенный фрагмент? Озеро Фушиан Ху, НАХОДЯЩЕЕСЯ В ГОРАХ КИТАЙСКОЙ ПРОВИНЦИИ ЮННАНЬ, тектонического происхождения.

19)Среди предложений 2–6 найдите предложение(-я) с деепричастным(-ыми) оборотом(-ами). Напишите номер(-а) этого(-их) предложения(-ий).

Другие тренировочные варианты по русскому языку 8 класс:

Ответы ВПР 2021 по русскому языку 8 класс все реальные варианты

ПОДЕЛИТЬСЯ МАТЕРИАЛОМ

Исправить нарушения в сочетаемости слов ⇐ Высшее образование

Информация
Уважаемые студенты!
Вместе с заданиями размещайте и свои варианты решения. В противном случае ваше сообщение будет проигнорировано.

Автор темы

львица

помощник писаря
помощник писаря
Всего сообщений: 3
Зарегистрирован: 15.02.2013
Образование: высшее гуманитарное
Профессия: финансист
Откуда: владимир
Возраст: 32

 

Исправить нарушения в сочетаемости слов

Здравствуйте! Помогите пожалуйста, исправить нарушения в сочетаемости слов:
1. играть большое значение — играет большое значение
2. вручить цветы — вручать цветы
3. назрела сложная обстановка — назреваеет сложная обстановка
4. придавать большое внимание —
5. разъяснения о допущенных ошибках —
6. оперировать с точными данными — оперирует точными данными
7. подчеркнуть о необходимости строительства — подчеркнуть необходимость строительства
8. сдать рецензию о книге —
9. согласно действующего приказа — действовать согласно приказу

Аватара пользователя

vadim_i_z

Гениалиссимус
Гениалиссимус
Всего сообщений: 7814
Зарегистрирован: 27.09.2006
Образование: высшее естественно-научное
Откуда: Минск, Беларусь
Возраст: 66

 

Re: Исправить нарушения в сочетаемости слов

Сообщение

vadim_i_z » 15 фев 2013, 12:10

Информация

Уважаемые студенты!
Вместе с заданиями размещайте и свои варианты решения. В противном случае ваше сообщение будет проигнорировано.

Аватара пользователя

Марго

Гениалиссимус
Гениалиссимус
Всего сообщений: 13772
Зарегистрирован: 11.11.2009

Лучшие Ответы: 2

Образование: высшее гуманитарное
Откуда: Моcква

 

Re: Исправить нарушения в сочетаемости слов

Сообщение

Марго » 15 фев 2013, 12:13

львица:6. оперировать с точными данными — оперирует точными данными

А зачем Вы меняете форму глагола? Это лишнее. Оперировать точными данными.

Добавлено спустя 1 минуту 47 секунд:

львица:7. подчеркнуть о необходимости строительства — подчеркнуть необходимость строительства

Вот это верно. Над остальным Вам надо еще подумать. Попытайтесь.

Автор темы

львица

помощник писаря
помощник писаря
Всего сообщений: 3
Зарегистрирован: 15.02.2013
Образование: высшее гуманитарное
Профессия: финансист
Откуда: владимир
Возраст: 32

 

Re: Исправить нарушения в сочетаемости слов

Сообщение

львица » 15 фев 2013, 12:22

сдать рецензию о книге — сдать рецензию книги

Добавлено спустя 2 минуты 2 секунды:
разъяснения о допущенных ошибках — разъяснения к допущенным ошибкам
согласно действующего приказа — согласно с действующим приказом

Аватара пользователя

Марго

Гениалиссимус
Гениалиссимус
Всего сообщений: 13772
Зарегистрирован: 11.11.2009

Лучшие Ответы: 2

Образование: высшее гуманитарное
Откуда: Моcква

 

Re: Исправить нарушения в сочетаемости слов

Сообщение

Марго » 15 фев 2013, 12:36

львица:сдать рецензию о книге — сдать рецензию книги

Сдать рецензию на книгу.

львица:согласно действующего приказа — согласно с действующим приказом

Согласно действующему приказу.

львица:разъяснения о допущенных ошибках — разъяснения к допущенным ошибкам

Я бы сказала: разъяснения по поводу допущенных ошибок.

Автор темы

львица

помощник писаря
помощник писаря
Всего сообщений: 3
Зарегистрирован: 15.02.2013
Образование: высшее гуманитарное
Профессия: финансист
Откуда: владимир
Возраст: 32

 

Re: Исправить нарушения в сочетаемости слов

Сообщение

львица » 15 фев 2013, 13:10

играть большое значение — имееть большое значение
вручить цветы — дарить цветы
назрела сложная обстановка — сложилась сложная обстановка
придавать большое внимание — придавать большое значение

H_N

лауреат и орденоносец
лауреат и орденоносец
Всего сообщений: 994
Зарегистрирован: 14.03.2012
Образование: высшее естественно-научное

 

Re: Исправить нарушения в сочетаемости слов

Сообщение

H_N » 15 фев 2013, 13:27

назрела сложная обстановка

Полагаю, это пример неудачного задания. Все три слова допускают замену; при этом образуемые словосочетания более или менее выдержат оценку на устойчивое согласование. А?

Аватара пользователя

Марго

Гениалиссимус
Гениалиссимус
Всего сообщений: 13772
Зарегистрирован: 11.11.2009

Лучшие Ответы: 2

Образование: высшее гуманитарное
Откуда: Моcква

 

Re: Исправить нарушения в сочетаемости слов

Сообщение

Марго » 15 фев 2013, 13:47

львица:играть большое значение — имееть большое значение
вручить цветы — дарить цветы
назрела сложная обстановка — сложилась сложная обстановка
придавать большое внимание — придавать большое значение

Чтобы избежать повторения слож- слож-, лучше, к примеру, создалась сложная обстановка. Остальное верно, только поправьте слово иметь. И в последнем можно написать уделять большое внимание, поскольку с большим значением у нас пример уже есть.

Для отправки ответа, комментария или отзыва вам необходимо авторизоваться

  • 1 Ответы
    1290 Просмотры
    Последнее сообщение Марго

    16 ноя 2010, 13:45

  • 14 Ответы
    2573 Просмотры
    Последнее сообщение Алексей-18

    15 ноя 2012, 11:08

  • 6 Ответы
    2403 Просмотры
    Последнее сообщение mary

    04 дек 2011, 21:51

  • 4 Ответы
    2971 Просмотры
    Последнее сообщение stasja

    31 янв 2011, 22:50

  • 2 Ответы
    2853 Просмотры
    Последнее сообщение Patriot Хренов

    01 окт 2015, 04:19

Презентация «Лексические ошибки»

В презентации представлена классификация лексических ошибок к новому варианту ЕГЭ 2018г.

Содержимое разработки

Лексические ошибки Автор: учитель русского языка и литературы МБОУ СОШ  с. Тополево Семенова Н. Н.

Лексические ошибки

Автор: учитель русского языка и литературы МБОУ СОШ

с. Тополево Семенова Н. Н.

Найти ошибки, исправить. 1. Подытоживая эти выводы, хочу обратить ваше внимание на следующие моменты. 2. Я хотел бы выразить слова благодарности организаторам этой выставки. 3.  Петр I ввел христианство, так как разноязыческая религия не давала единого сплочения. 4. Две школы разработали план совместного сотрудничества.   5. К недостаткам работы можно отнести недостаточное количество иллюстративного материала .

Найти ошибки, исправить.

  • 1. Подытоживая эти выводы, хочу обратить ваше внимание на следующие моменты.
  • 2. Я хотел бы выразить слова благодарности организаторам этой выставки.
  • 3.  Петр I ввел христианство, так как разноязыческая религия не давала единого сплочения.
  • 4. Две школы разработали план совместного сотрудничества.  
  • 5. К недостаткам работы можно отнести недостаточное количество иллюстративного материала .

Виды лексических ошибок Неразличение паронимов; Лексическая несочетаемость; Ошибочное употребление иноязычных слов; Плеоназм; Тавтология; Ошибки, связанные с неполнотой высказывания; Неуместное употребление стилистически сниженной лексики; Употребление слова в несвойственном ему значении.

Виды лексических ошибок

  • Неразличение паронимов;
  • Лексическая несочетаемость;
  • Ошибочное употребление иноязычных слов;
  • Плеоназм;
  • Тавтология;
  • Ошибки, связанные с неполнотой высказывания;
  • Неуместное употребление стилистически сниженной лексики;
  • Употребление слова в несвойственном ему значении.

Неразличение паронимов Письмо отправили  адресанту по почте. Человек ведет праздничную жизнь. Заглавный герой романа «12 стульев» никогда не терял чувства юмора. Этот человек был полный невежа в вопросах искусства. Как только предоставится возможность, я поеду на юг.

Неразличение паронимов

  • Письмо отправили  адресанту по почте.
  • Человек ведет праздничную жизнь.
  • Заглавный герой романа «12 стульев» никогда не терял чувства юмора.
  • Этот человек был полный невежа в вопросах искусства.
  • Как только предоставится возможность, я поеду на юг.

Лексическая несочетаемость Хороший руководитель должен во всем показывать образец своим подчиненным. Два единственных вопроса тревожили жителей города: вода и тепло. Внеклассная работа играет положительное значение в развитии детей. Подавляющее количество заданий было выполнено. В голове у него метнулась мысль о побеге.

Лексическая несочетаемость

  • Хороший руководитель должен во всем показывать образец своим подчиненным.
  • Два единственных вопроса тревожили жителей города: вода и тепло.
  • Внеклассная работа играет положительное значение в развитии детей.
  • Подавляющее количество заданий было выполнено.
  • В голове у него метнулась мысль о побеге.

Ошибочное употребление иноязычных слов Девушка конфиденциально призналась подругам, что переменила имя Катя на Кармен, потому что последнее импонирует её внешности. Презентация парфюма состоялась в прошлую пятницу. Срок выполнения заказа может быть пролонгирован. Нас попросили репрезентовать некоторые материалы. Дефекты в подготовке выпускников вскроются во время экзаменов.

Ошибочное употребление иноязычных слов

  • Девушка конфиденциально призналась подругам, что переменила имя Катя на Кармен, потому что последнее импонирует её внешности.
  • Презентация парфюма состоялась в прошлую пятницу.
  • Срок выполнения заказа может быть пролонгирован.
  • Нас попросили репрезентовать некоторые материалы.
  • Дефекты в подготовке выпускников вскроются во время экзаменов.

Плеоназм Он крепко держит в своих руках штурвал руля. Все гости получили памятные сувениры. Эта девочка оставила о себе очень прекрасное впечатление. О моей автобиографии я уже рассказывал во вступительной статье. Больной был немедленно госпитализирован в больницу.

Плеоназм

  • Он крепко держит в своих руках штурвал руля.
  • Все гости получили памятные сувениры.
  • Эта девочка оставила о себе очень прекрасное впечатление.
  • О моей автобиографии я уже рассказывал во вступительной статье.
  • Больной был немедленно госпитализирован в больницу.

Тавтология Руководители предприятий настроены на деловой настрой. Пилот вынужден был совершить вынужденную посадку в проливе Ла-Манш. Но ведь вскрываются все новые факты беззаконных деяний различных деятелей. Активисты активно участвуют в работе с молодежью. В конце шестидесятых годов сложилась сложная ситуация.

Тавтология

  • Руководители предприятий настроены на деловой настрой.
  • Пилот вынужден был совершить вынужденную посадку в проливе Ла-Манш.
  • Но ведь вскрываются все новые факты беззаконных деяний различных деятелей.
  • Активисты активно участвуют в работе с молодежью.
  • В конце шестидесятых годов сложилась сложная ситуация.

Ошибки, связанные с неполнотой высказывания Достоинство Куприна в том, что ничего лишнего. Наши дворники обязаны были посыпать песком тротуары, мосты и другие, что в их ведении. На уроках ребята хорошо ведут. Необходимо повысить эффективность. Я уже чувствую после болезни хорошо

Ошибки, связанные с неполнотой высказывания

  • Достоинство Куприна в том, что ничего лишнего.
  • Наши дворники обязаны были посыпать песком тротуары, мосты и другие, что в их ведении.
  • На уроках ребята хорошо ведут.
  • Необходимо повысить эффективность.
  • Я уже чувствую после болезни хорошо

Неуместное употребление стилистически сниженной лексики Эта история вполне тянет на сюжет для фильма-боевика. Куприн – потрясающий писатель своего времени. Есть большое желание заполучить очередное издание детектива. Этот фильм оказался абсолютно плоским боевиком. Хорошие ученики завсегда выполняют домашние задания

Неуместное употребление стилистически сниженной лексики

  • Эта история вполне тянет на сюжет для фильма-боевика.
  • Куприн – потрясающий писатель своего времени.
  • Есть большое желание заполучить очередное издание детектива.
  • Этот фильм оказался абсолютно плоским боевиком.
  • Хорошие ученики завсегда выполняют домашние задания

Употребление слова в несвойственном ему значении Костер все больше и больше распалялся, пылал. Любителям холодных коктейлей смешивают те же компаньоны, но в других пропорциях. Благодаря пожару, вспыхнувшему от костра, сгорел большой участок леса. Докладчики обычно фигурируют такими выражениями, как «имеет место», «оказывает помощь» и т.п. Перед началом учебы я обратно прочел рассказы А.П. Чехова.

Употребление слова в несвойственном ему значении

  • Костер все больше и больше распалялся, пылал.
  • Любителям холодных коктейлей смешивают те же компаньоны, но в других пропорциях.
  • Благодаря пожару, вспыхнувшему от костра, сгорел большой участок леса.
  • Докладчики обычно фигурируют такими выражениями, как «имеет место», «оказывает помощь» и т.п.
  • Перед началом учебы я обратно прочел рассказы А.П. Чехова.

Вместо точек вставьте слова, которые наиболее точно выражают мысль; мотивируйте свой выбор. Человек … (изобрел, нашел, отыскал, придумал, создал) слова для всего, что обнаружено им … (в мире, во вселенной, на земле). Но этого мало. Он … (назвал, объяснил, определил, указал на) всякое действие и состояние. Он … (назвал, обозначил, объяснил, окрестил, определил) словами свойства и качества всего, что его окружает. Словарь … (воспроизводит, определяет, отображает, отражает, фиксирует) все изменения, … (происходящие, совершающиеся, существующие) в мире. Он … (запечатлел, отразил, сохранил) опыт и мудрость веков и, не отставая, сопутствует жизни, … (движению, прогрессу, развитию) техники, науки, искусства. Он может … (выделить, назвать, обозначить, определить, указать на) любую вещь и располагает средствами для … (выражения, обозначения, объяснения, передачи, сообщения) самых отвлеченных и обобщенных идей и понятий

Вместо точек вставьте слова, которые наиболее точно выражают мысль; мотивируйте свой выбор.

  • Человек … (изобрел, нашел, отыскал, придумал, создал) слова для всего, что обнаружено им … (в мире, во вселенной, на земле). Но этого мало. Он … (назвал, объяснил, определил, указал на) всякое действие и состояние. Он … (назвал, обозначил, объяснил, окрестил, определил) словами свойства и качества всего, что его окружает. Словарь … (воспроизводит, определяет, отображает, отражает, фиксирует) все изменения, … (происходящие, совершающиеся, существующие) в мире. Он … (запечатлел, отразил, сохранил) опыт и мудрость веков и, не отставая, сопутствует жизни, … (движению, прогрессу, развитию) техники, науки, искусства. Он может … (выделить, назвать, обозначить, определить, указать на) любую вещь и располагает средствами для … (выражения, обозначения, объяснения, передачи, сообщения) самых отвлеченных и обобщенных идей и понятий

Проверьте себя Человек нашел слова для всего, что обнаружено им во вселенной . Но этого мало. Он назвал всякое действие и состояние. Он определил словами свойства и качества всего, что его окружает. Словарь отражает все изменения, происходящие в мире. Он запечатлел опыт и мудрость веков и, не отставая, сопутствует жизни, развитию техники, науки, искусства. Он может назвать любую вещь и располагает средствами для выражения самых отвлеченных и обобщенных идей и понятий.

Проверьте себя

  • Человек нашел слова для всего, что обнаружено им во вселенной . Но этого мало. Он назвал всякое действие и состояние. Он определил словами свойства и качества всего, что его окружает. Словарь отражает все изменения, происходящие в мире. Он запечатлел опыт и мудрость веков и, не отставая, сопутствует жизни, развитию техники, науки, искусства. Он может назвать любую вещь и располагает средствами для выражения самых отвлеченных и обобщенных идей и понятий.

Нарушения логики высказывания используются как стилистический приём создания комического эффекта. 1) Как только я выдержала экзамены, то сейчас же поехала с маменькой, мебелью  и  братом… на дачу.      (А.П.Чехов)    / Отсутствие логики/ 2)  Взять жену без состояния я в состоянии,  но входить в долги из-за тряпок я не в состоянии.  (А.С.Пушкин).  / Каламбур, словесная шутка /   3) Вдруг какой – нибудь  эдакой, можете себе представить себе, Невский проспект, или там, знаете, какая – нибудь  Гороховая,  чёрт возьми, или там эдакая какая – нибудь Литейная;  там шпиц эдакой какой – нибудь  в  воздухе; мосты там висят эдаким чёртом…   ( Н.В. Гоголь ).   / Повтор слов, ругательные слова/.

Нарушения логики высказывания используются как стилистический приём создания комического эффекта.

  • 1) Как только я выдержала экзамены, то сейчас же поехала с маменькой, мебелью  и  братом… на дачу.      (А.П.Чехов)    / Отсутствие логики/
  • 2)  Взять жену без состояния я в состоянии,  но входить в долги из-за тряпок я не в состоянии.  (А.С.Пушкин).  / Каламбур, словесная шутка /
  •   3) Вдруг какой – нибудь  эдакой, можете себе представить себе, Невский проспект, или там, знаете, какая – нибудь  Гороховая,  чёрт возьми, или там эдакая какая – нибудь Литейная;  там шпиц эдакой какой – нибудь  в  воздухе; мосты там висят эдаким чёртом…   ( Н.В. Гоголь ).   / Повтор слов, ругательные слова/.

Любил студентов засыпать он, видно, оттого, Что те любили засыпать на лекциях его (Марш.). Народ был, народ есть, народ будет есть. Защитник вольности и прав в сем случае совсем не прав (П.).

  • Любил студентов засыпать он, видно, оттого, Что те любили засыпать на лекциях его (Марш.).
  • Народ был, народ есть, народ будет есть.
  • Защитник вольности и прав в сем случае совсем не прав (П.).

Ведь ваши принципы просты: Вы очень любите остроты. Но вы боитесь остроты (А. Безыменский. Эпиграмма «Многим журналам, издательствам, редакторам»). Одно, брат, дело - огурцов посол. Другое - если ты посол (Козл.) Но свою неправую правую не сменю на правую левую (Выс. «Песня про прыгуна в высоту»).

  • Ведь ваши принципы просты: Вы очень любите остроты. Но вы боитесь остроты (А. Безыменский. Эпиграмма «Многим журналам, издательствам, редакторам»).
  • Одно, брат, дело — огурцов посол. Другое — если ты посол (Козл.)
  • Но свою неправую правую не сменю на правую левую (Выс. «Песня про прыгуна в высоту»).

Исправьте ошибки     1. Не стоит быть даже искушенным читателем, чтобы понять, насколько это произведение далеко от совершенства 2. Поэма, напечатанная вместе с «Мойдодыром» и «Тараканищем», стала одним из самых любимых произведений юных читателей 3. Писатель извлекает своих героев из самой действительности 4. Улицы в нашем городе протяженные, пешком ходить по ним трудно 5. У нас имеется несколько охотничьих заказников, которые обыкновенно относят к разряду особо охраняемых территорий. 6. Новый завод был богат новой техникой, требующей деликатного обращения 7. Этого успеха удалось достичь благодаря повышению применения средств авиации и повышению технического оснащения предприятий 8.  Читая роман, мысленно окунаешься в жизнь патриархальной русской семьи

Исправьте ошибки

  •     1. Не стоит быть даже искушенным читателем, чтобы понять, насколько это произведение далеко от совершенства
  • 2. Поэма, напечатанная вместе с «Мойдодыром» и «Тараканищем», стала одним из самых любимых произведений юных читателей
  • 3. Писатель извлекает своих героев из самой действительности
  • 4. Улицы в нашем городе протяженные, пешком ходить по ним трудно
  • 5. У нас имеется несколько охотничьих заказников, которые обыкновенно относят к разряду особо охраняемых территорий.
  • 6. Новый завод был богат новой техникой, требующей деликатного обращения
  • 7. Этого успеха удалось достичь благодаря повышению применения средств авиации и повышению технического оснащения предприятий
  • 8.  Читая роман, мысленно окунаешься в жизнь патриархальной русской семьи

    1. Не нужно даже быть слишком взыскательным читателем, чтобы понять, как далеко это произведение от подлинного искусства     2. Опубликованная одновременно с «Мойдодыром» и «Тараканищем» поэма стала одним из самых любимых произведений ребят     3. Писатель берет своих героев из жизни.     4. Улицы в нашем городе длинные, ходить по ним нелегко. 5. У нас есть несколько заповедников, которые обычно относят к числу особо охраняемых территорий 6. Новый завод был оснащен современной техникой, требующей умелого обращения 7. Этого успеха удалось добиться благодаря широкому применению возможностей авиации и увеличению технической оснащенности предприятий     8. Читая роман, мысленно погружаешься в жизнь патриархальной русской семьи

  •     1. Не нужно даже быть слишком взыскательным читателем, чтобы понять, как далеко это произведение от подлинного искусства
  •     2. Опубликованная одновременно с «Мойдодыром» и «Тараканищем» поэма стала одним из самых любимых произведений ребят
  •     3. Писатель берет своих героев из жизни.
  •     4. Улицы в нашем городе длинные, ходить по ним нелегко.
  • 5. У нас есть несколько заповедников, которые обычно относят к числу особо охраняемых территорий
  • 6. Новый завод был оснащен современной техникой, требующей умелого обращения
  • 7. Этого успеха удалось добиться благодаря широкому применению возможностей авиации и увеличению технической оснащенности предприятий
  •     8. Читая роман, мысленно погружаешься в жизнь патриархальной русской семьи

Спасибо за внимание!

Спасибо за внимание!



-82%

Похожие файлы

  • Урок русского языка «Группы слов по значению. Синонимы»

  • Презентация к уроку русского языка «Буквы О и Е в окончаниях имён существительных после шипящих и Ц»

  • Массивы

  • Урок русского языка по теме: «Ы-и после ц»

  • Конспект урока и презентация по русскому языку «Слова исконно русские и заимствованные»

Вы смотрели

Кто не делится найденным, подобен свету в дупле секвойи (древняя индейская пословица)

Библиографическая запись:
Типичные случаи нарушения синтаксических норм письменной речи и задачи корректора и редактора по их устранению. — Текст : электронный // Myfilology.ru – информационный филологический ресурс : [сайт]. – URL: https://myfilology.ru//168/tipichnye-sluchai-narusheniya-sintaksicheskix-norm-pismennoj-rechi-i-zadachi-korrektora-i-redaktora-po-ix-ustraneniyu/ (дата обращения: 7.06.2023)

Синтаксические нормы определяют правила построения основных синтаксических единиц, словосочетаний и предложений. Эти нормы включают правила согласования слов и синтаксического управления, соотнесения частей предложения друг с другом с помощью грамматических форм слов для того, чтобы предложение было грамотным и осмысленным.

Случаи нарушения синтаксических норм

  • неправильное согласование (Группа запорожцев написали письмо султану);
  • неправильное управление (Мы действуем согласно распоряжения),
  • местоименное дублирование подлежащего (Мама она хорошо готовит);
  • ошибки в конструировании сказуемого (Я предан Родине и беспощадный к её врагам);
  • отсутствие видо-временной соотносительности глаголов-сказуемых (Я читаю книгу и представила её персонажей);
  • морфологическая неоднородность однородных членов (Книги помогают нам хорошо учиться и в выборе профессии);
  • неправильный порядок слов (Большие и красивые на окраине города здания вырастали быстро);
  • двойная синтаксическая связь (Жильцы требовали устранения неполадок и ремонта);
  • синтаксическая двузначность (Воровство детей);
  • «свободный» деепричастный оборот (Играя на улице, развязался шнурок);
  • установление отношений однородности между членами простого предложения и частями сложного (Он ждал темноты и когда все уснут);
  • смещение конструкции (Первое, о чем я вас прошу, это о внимании);
  • смешение прямой и косвенной речи (Базаров говорил, что мой дед землю пахал);
  • неправильный выбор союза (Я прочитал статью, где говорится о достижениях современной медицины);
  • одновременное использование подчинительного и сочинительного союзов (Когда Дубровского посадили в клетку с медведем, и он не побоялся и убил медведя);
  • дублирование союзов (Ты сказал, что будто бы видел этот фильм);
  • неправильная расстановка частей составного союза (Мы собрали не только много грибов, но и поймали белку);
  • разноструктурность однородных частей бессоюзного сложного предложения (Об ответе Петрова можно сказать следующее: ответ не соответствует плану;

Можно отдельно выделить синтаксические ошибки на уровне словосочетания (например, неправильное согласование, неправильное управление и др.), на уровне простого предложения (например, отсутствие видо-временной соотнесенности глаголов-сказуемых и др.) и на уровне сложного предложения (например, разноструктурность однородных частей бессоюзного сложного предложения и др.).

Задачи корректора и редактора по устранению синтаксических ошибок

Редактор должен представлять, как происходит понимание текста, как влияет на этот процесс выбор слова или синтаксической конструкции. Нужно хорошо знать все тонкости значений слов, чтобы подобрать нужный синоним. Следует понимать, какие словосочетания и конструкции тяжелы для восприятия. Но главное — необходимо владеть нормами литературного языка, а это непростая задача, хотя на помощь постоянно приходят справочники.

Русский язык отличается удивительным разнообразием синтаксических конструкций. В зависимости от жанра, темы, характера материала журналист и редактор всегда могут выбрать нужную синтаксическую конструкцию. И не следует думать, что экспрессивная окраска речи свойственна лишь разговорному синтаксису и возможна только в сниженной речи. Известно немало приемов усиления выразительности той или иной формулировки синтаксическими средствами. Подтвердим эту мысль еще несколькими примерами синтаксической синонимии: Я читаю книгу с удовольствием. — Читаю книгу с удовольствием. — Книга! Ее нельзя читать без удовольствия. — Читая книгу, получаешь удовольствие. — Книга читается с удовольствием. — Книгу читают с удовольствием. — Мне сегодня читается: книга интересная. — Я не могу читать книгу, не получая удовольствия и т.д. 

Для автора и редактора изучение синтаксической синонимии не менее важно, чем освоение синонимических богатств лексики. Но если в работе над лексикой можно обратиться к синонимическим словарям, то при исправлении синтаксических конструкций приходится ориентироваться лишь на собственное знание грамматики родного языка. Поэтому стилистическая характеристика синтаксической синонимии представляет особый интерес для тех, кто работает над стилем рукописи.

Нередко синонимичны и разные типы односоставных предложений, например определенно-личные — безличные: Дыши последней свободой (A.A. Ахматова). — Надо дышать последней свободой, неопределенно-личные — безличные: Близким говорят правду. — Близким принято говорить правду, обобщенно-личные — безличные: Говори, да не заговаривайся (пословица). — Говорить можно, да не надо заговариваться; номинативные — безличные: Тишина. — Тихо; Озноб, лихорадка. — Знобит, лихорадит; инфинитивные — безличные: Не нагнать тебе бешеной тройки (H.A. Некрасов). — Невозможно нагнать тебе бешеную тройку.

В неопределенно-личных предложениях подчеркивается действие: Подсудимых куда-то выводили и только что ввели назад (Л.Н. Толстой); Сейчас за вами придут (K.M. Симонов). Употребление таких предложений позволяет акцентировать внимание на глаголе-сказуемом, в то время как субъект действия отодвигается на задний план независимо от того, известен он говорящим или нет.

Безличные предложения нередко трансформируются в двусоставные или односоставные неопределенно- или определенно-личные. Ср.: Сегодня тает. — Снег тает; Следы засыпало снегом. — Следы засыпал снег; Метет. — Метет пурга; Хочется есть. — Я хочу есть; Где тебя носило? — Где ты был?; Следует уступать место старшим. — Уступайте место старшим; Полагается принимать лекарство. — Принимайте лекарство; Меня там не было. — Я там не был.

При возможности двоякого выражения мысли следует учитывать, что личные конструкции содержат элемент активности, проявления воли действующего лица, уверенности в совершении действия, тогда как безличным оборотам присущ оттенок пассивности, инертности.

Инфинитивные предложения предоставляют значительные возможности для эмоционального и афористического выражения мысли: Чему быть, того не миновать (пословица); Кого любить, кому же верить? (М.Ю. Лермонтов); Так держать/ От судьбы не уйти. Они используются в пословицах, в художественной речи; эта конструкция приемлема для лозунгов: Работать без брака! Однако основная сфера их функционирования — разговорная речь: Сказать бы об этом сразу! А не вернуться ли нам? Берега не видать. Последняя конструкция, распространенная дополнением, имеет просторечную окраску. Экспрессивность препятствует использованию инфинитивных конструкций в книжных стилях.

Номинативные предложения по сути своей как бы созданы для описания: в них заложены большие изобразительные возможности. Называя предметы, расцвечивая их определениями, литераторы рисуют картины природы, обстановку, описывают состояние героя, дают оценку окружающему миру: Открытое пространство большого аэродрома, залитого солнцем. Грандиозная перспектива самолетов, выстроенных к параду. Оживленные группы военных летников. Эту картину нужно видеть собственными глазами… (Из газет) Однако подобные описания указывают лишь на бытие и не способны нарисовать развитие действия. Даже если номинативы — отглагольные существительные и с помощью их рисуется живая картина, то и в этом случае они позволяют запечатлеть одно мгновение, один кадр: Бой барабанный, клики, скрежет, гром пушек, топот, ржанье, стон… (A.C. Пушкин); Смятенье! обморок! поспешность! гнев! испуга!.. (A.C. Грибоедов) Линейное описание событий номинативными предложениями невозможно: они фиксируют только настоящее время.

Стилистические возможности русского синтаксиса расширяются и благодаря тому, что с полными предложениями могут успешно конкурировать предложения неполные, имеющие яркую экспрессивную окраску. Их стилистическое использование в речи определяет грамматическая природа этих предложений.

Неполные предложения, образующие диалогические единства, создаются непосредственно в процессе живого общения: — Когда ты придешь? — Завтра. — Одна или с Виктором? — Конечно, с Виктором. Из разговорной речи они проникают в художественную и публицистическую как характерная особенность диалога: — Прекрасный вечер, — начал он, — так тепло!Вы давно гуляете?— Нет, недавно (И.С. Тургенев).

Иными мотивами обусловлено предпочтение эллиптически м предложениям (от греч. ellipsis — выпадение, опущение), т.е. таким, в которых опущен какой-либо член предложения, легко восстанавливаемый из контекста. Они выступают как сильное средство эмоциональности речи. Сфера их применения — разговорная речь, они нужны писателям и журналистам для построения диалога, передачи прямой речи. Эллиптические конструкции придают фразе особый динамизм: Я к ней, а он в меня раз из пистолета (А.Н. Островский); К барьеру! (А.П. Чехов); Назад, домой, на родину… (А.Н. Толстой). Как видим, русский синтаксис предоставляет в наше распоряжение самые различные конструкции. Их нужно умело и уместно использовать в речи. И тогда она будет яркой, богатой.

Стилистическая оценка параллельных синтаксических конструкций

Проиллюстрируем синтаксический параллелизм простыми примерами.

  1. 1. Редактор прочитал рукопись и написал рабочую рецензию.
  2. 2. Редактор, прочитав рукопись, написал рабочую рецензию.
  3. 3. Редактор, прочитавший рукопись, написал рабочую рецензию.
  4. 4. Редактор закончил чтение рукописи и приступил к написанию рабочей рецензии.
  5. 5. Когда редактор прочитал рукопись, он смог написать рабочую рецензию.
  6. 6. После прочтения рукописи редактор написал рабочую рецензию.
  7. 7. Написание рабочей рецензии редактором стало возможным после прочтения рукописи.

Как видим, эти конструкции отличаются главным образом способами обозначения действия, которые в русском языке различны. Чаще всего действие обозначает спрягаемая форма глагола (пример 1), но то же значение передает и деепричастие (пример 2), причастие (пример 3), отглагольное существительное в составе расщепленного сказуемого (пример 4), инфинитив (как часть составного глагольного сказуемого — пример 5), наконец, отглагольные существительные (примеры 6, 7). Все эти конструкции отличаются оттенками значения и стилистической окраской. Наиболее удобны нейтральные, спрягаемые формы глагола; причастия и деепричастия имеют книжную окраску; отглагольные существительные выделяются канцелярским звучанием, которое еще усиливается пассивной синтаксической конструкцией (пример 7). Наблюдаемое при этом «убывание глагольности» от примера к примеру сказывается на эстетической стороне речи: ее не украшают отглагольные существительные, они придают высказыванию тяжеловесность.

Из этого следует, что автор и редактор должны использовать параллельные синтаксические конструкции, учитывая характер текста, его жанр и общую стилевую установку в определенном произведении. Важно также иметь в виду оттенки значения глагольных слов в том или ином предложении. Так, в примерах 1 и 5 подчеркнуто главное действие (прочитал), сообщение о нем в придаточной части предложения (конструкция 5) способствует актуализации этой глагольной формы, которая при этом обретает большую самостоятельность.

Преобразование конструкции приводит к утрате глагольности, следовательно, в примерах 2, 3, а тем более — в конструкциях 4, 6, 7 действие отодвигается на второй план. Причастие и деепричастие еще более или менее удерживают глагольные признаки (значение вида придает высказыванию большую определенность, значение времени, свойственное только причастиям, также важно для характеристики действия). Но если деепричастный оборот («второе сказуемое») акцентирует внимание на дополнительном действии, то причастный оборот, выступающий как распространенное определение, усиливает, выделяет субъект действия (редактор, прочитавший рукопись, а не кто другой). Инфинитив, не обладающий важными глагольными признаками, только называет действие (смог написать рецензию), но не может дать полное о нем представление. И наконец, отглагольное существительное нейтрализует представление о действии как о динамическом процессе, указывая лишь на сам факт его совершения.

Таким образом, несмотря на богатство и разнообразие синтаксических конструкций в русском языке, их параллелизм — весьма условное понятие. Параллельные синтаксические конструкции должны употребляться с учетом их семантических и стилистических особенностей, ролью в общей структуре текста, и было бы заблуждением считать их равноценными и взаимозаменяемыми.

Остановимся более подробно на некоторых параллельных синтаксических конструкциях. Большие возможности для стилистического выбора предоставляет синонимия причастных оборотов и придаточных определительных частей сложноподчиненных предложений: Автор ознакомился с отзывом, написанным рецензентом. — Автор ознакомился с отзывом, который написал рецензент. Преимущество первой конструкции — в ее лаконизме, вторая же акцентирует внимание на действии, указанном в придаточной части сложного предложения; первая тяготеет к книжным стилям, вторая стилистически нейтральна. Из этого, однако, не следует, что для письменной речи всегда предпочтительнее причастный оборот. Он уступает в выразительности придаточной части, если по условиям контекста необходимо подчеркнуть значение действия: Снисхождения заслуживают лишь те люди, которые признают критику и исправляют свои ошибки (ср.: Снисхождения заслуживают лишь люди, признающие критику и исправляющие свои ошибки). Попутно заметим, что препозитивный причастный оборот в таких случаях наименее выразителен, поскольку он подчеркивает значение субъекта действия, а не само действие (ср.: Снисхождения заслуживают лишь признающие критику и исправляющие свои ошибки люди). Поэтому, заменяя причастным оборотом придаточную определительную часть сложноподчиненного предложения, не следует забывать, что это ведет к ослаблению глагольности и, следовательно, к умалению самостоятельного значения развернутого определения.

Множество параллельных синтаксических конструкций образуют деепричастные обороты и другие члены предложения, а также придаточные части сложноподчиненного предложения. Например: Беседуя с редактором, автор понял… — Во время беседы с редактором автор понял… Вернувшись из командировки, журналист зашел в редакцию — После возвращения из командировки… Нисколько не колеблясь, я принял условия — Без всяких колебаний я принял условия; Он говорил о достоинствах своего произведения, нисколько не смущаясь — Он говорил… и нисколько не смущался. В последнем примере глагол, заменивший деепричастие, подчеркнул значение действия; во всех же других примерах отглагольные существительные уменьшили весомость действия.

Напротив, действие будет акцентировано при замене деепричастного оборота придаточной частью сложноподчиненного предложения: Редактор несколько раз перечитывал абзац, стараясь разобраться в новой терминологии — …перечитывал абзац, чтобы разобраться… Я понимал, что, ввязавшись в этот спор, должен отстаивать свою точку зрения — Я понимал, что уж если ввязался в этот спор, должен отстаивать свою точку зрения; Редколлегия не рекомендовала статью к печати, получив отрицательный отзыв рецензента — …не рекомендовала статью к пeчати, потому что получила отрицательный отзыв рецензента. Все эти конструкции при близости значения имеют свои смысловые оттенки и грамматические особенности; союзы уточняют семантические нюансы. Однако деепричастные обороты формулируют мысль более экономно, хотя их отличает книжная окраска. В особых случаях деепричастный оборот избавляет от неясности высказывания. Например, нецелесообразно заменять придаточным деепричастный оборот в такой конструкции: Беседуя с журналистом, редактор понял — Когда редактор беседовал с журналистом, он понял (местоимение он воспринимается двояко: Кто понял — редактор или журналист?).

Синтаксический параллелизм характеризует и конструкции, образующие пары сложноподчиненных предложений и предложений с инфинитивом или отглагольным существительным. Например:

Инфинитивные конструкции отличаются от синонимических придаточных частей сложноподчиненных предложений большей категоричностью и привлекают лаконизмом. В сложноподчиненных предложениях подчеркнуто действие, а при использовании личной формы глагола актуализации подвергается субъект действия. Поэтому предпочтение той или иной конструкции зависит от условий контекста.

Конструкции с отглагольными существительными имеют книжную, а нередко и канцелярскую окраску, но их преимущество — лаконизм, поэтому они широко используются в научном и официально-деловом стилях. При литературном редактировании текстов, в которых скопление отглагольных существительных создает неудобства, целесообразно соблюдать чувство меры, чередуя употребление синонимических конструкций.

Разнообразие инфинитивных конструкций предоставляет богатый материал для стилистического выбора. При этом важно учитывать семантические и стилистические оттенки синонимических конструкций. Рассмотрим некоторые случаи.

При переносном употреблении инфинитив обычно получает в контексте значение лица и воспринимается как указание на действие, происходящее реально во времени: Дрозд горевать, дрозд тосковать (И.А. Крылов). Синонимическими конструкциями могут быть такие, в которых употреблены глаголы изъявительного наклонения прошедшего времени совершенного вида: дрозд загоревал, дрозд затосковал или глаголы в настоящем времени, выступающем в значении прошедшего: дрозд горюет, дрозд тоскует. Сопоставляя подобные синонимические конструкции, легко заметить, что инфинитив передает действие более интенсивное. А это значит, что при переносном употреблении неопределенная форма глагола становится ярким средством речевой экспрессии. Добавление к инфинитиву частицы ну подчеркнет начало действия и усилит его интенсивность: Отколе ни возьмись, навстречу Моська им. Увидевши Слона, ну на него метаться, и лаять, и визжать, и рваться… (И.А. Крылов)

Сильным источником увеличения действенности речи является переносное употребление инфинитива в значении повелительного и сослагательного наклонения: Молчать! Немедленно ехать! И — Ах, скорее бы, уйти! (А.П. Чехов); Стать бы волшебником! В таких случаях инфинитивная конструкция оказывается незаменимым средством выражения сильного стремления говорящего к совершению какого-то действия. Замена инфинитивных конструкций ведет к потере экспрессивной окраски речи.

Именно инфинитивная конструкция с отрицанием убедительно выражает полную невозможность действия. Таким образом, отвлеченность инфинитива, отсутствие у него конкретных грамматических категорий создают условия для употребления этого, по выражению A.M. Пешковского, «голого» носителя «идеи действия», как более выразительного эквивалента разнообразных глагольных форм в экспрессивной речи.

Использование параллельных синтаксических конструкций при стилистической правке

Разнообразие синтаксических вариантов выражения мысли, обилие параллельных конструкций в русском языке создает широкие возможности для выбора языковых средств при литературном редактировании текста на синтаксическом уровне. Рассмотрим наиболее распространенные приемы стилистической правки синтаксических конструкций редактором.

Анализируя употребление в речи причастных оборотов, редактор часто замечает ошибки в образовании причастий и, устраняя их, заменяет эту конструкцию придаточным определительным предложением. Рассмотрим примеры такой стилистической правки:

В первом примере причастие образовано от глагола сослагательного наклонения; во втором причастие настоящего времени — от глагола совершенного вида, в третьем — страдательное причастие от непереходного глагола; в четвертом — действительное причастие — от возвратного глагола неправомерно заменило страдательное выполняемый. Все эти нарушения языковой нормы редактор устранил, заменив причастные обороты параллельными синтаксическими конструкциями. Подобная же стилистическая правка позволяет избежать нанизывания причастных оборотов, делающих предложение громоздким и тяжеловесным.

Встречаются ошибки, связанные с нарушением порядка слов в причастном обороте: Приехавшие делегаты на конференцию должны зарегистрироваться — определяемое слово (делегаты) оказалось внутри причастного оборота. Возможна такая стилистическая правка: Приехавшие на конференцию делегаты должны зарегистрироваться. Или: Делегаты, приехавшие на конференцию; наконец: Делегаты, которые приехали на конференцию, должны зарегистрироваться.

Недопустим разнобой видо-временных форм глагола-сказуемого и причастия, за этим также должен следить редактор. Стилистическая правка в этом случае сводится к приведению в соответствие глагольных форм/

Использование параллельных синтаксических конструкций помогает устранить ошибки и при неправильном употреблении деепричастных оборотов. Иногда деепричастный оборот употребляется «самостоятельно» — при отсутствующем подлежащем (в безличном предложении) или относится к другому субъекту действия, чем глагол-сказуемое: Прочитав внимательно рукопись, редактору стало ясно, что работы с ней будет много; Убежав из чеченского плена, солдата вскоре нашла мать. Такие предложения нужно исправлять: Когда редактор прочитал рукопись, ему стало ясно… Или: Прочитав внимательно рукопись, редактор понял… Солдат убежал из чеченского плена, и вскоре его нашла мать. Или: Солдата, убежавшего из чеченского плена, вскоре нашла мать.

Использование параллельных синтаксических конструкций значительно облегчает стилистическую правку текстов, в которых допущены ошибки в построении предложений, а также в случаях устранения нежелательной канцелярской окраски речи. 

Стилистическая оценка предложений простых и сложных

Первое, что приходит на ум при постановке вопроса об использовании простых и сложных предложений, — это проблема длины синтаксической конструкции. Что лучше — предложение длинное или короткое?

Ответ не так прост, как может показаться. В устной речи мы, конечно, предпочитаем короткие фразы, в книжных стилях господствуют длинные, многочленные предложения с различными видами сочинительной и подчинительной связи. Такие конструкции могут показаться тяжеловесными и затруднят восприятие текста. Однако было бы глубоким заблуждением считать, что автор должен всегда выбирать короткие, «легкие» фразы. Если бы во всех случаях лучшим способом выражения мысли были короткие простые предложения, язык бы уже давно освободился от больших, громоздких фраз, их избегали бы журналисты и писатели. У А.П. Чехова мы не нашли бы ни одного сложноподчиненного предложения, ведь он утверждал: «Краткость — сестра таланта»! Но он умел строить и большие, длинные предложения. Вспомним одно такое из рассказа «Человек в футляре»:

А на педагогических советах он просто угнетал нас своею осторожностью, мнительностью и своими чисто футлярными соображениями насчет того, что вот-де в мужской и женской гимназиях молодежь ведет себя дурно, очень шумит в классах, — ах, как бы не дошло до начальства, ах, как бы чего не вышло, — и что если б из второго класса исключить Петрова, а из четвертого Егорова, то было бы очень хорошо.

Мастером стилистического использования сложных синтаксических конструкций был Л.Н. Толстой, и А.П. Чехов нашел эстетическое обоснование приверженности знаменитого романиста усложненному синтаксису. С.Н. Щукин вспоминал о замечании А.П. Чехова: «Вы обращали внимание на язык Толстого? Громадные периоды, предложения нагромождены одно на другое. Не думайте, что это случайно, что это недостаток. Это искусство, и оно дается после труда. Эти периоды производят впечатление силы». Художественная речь Толстого отражает его сложный, глубинный анализ изображаемой жизни, и писателю важно показать не результат своих наблюдений, а сам поиск истины.

Интересна и оценка длины предложения A.M. Горьким, стиль которого отличает также усложненный синтаксис. Одному из начинающих авторов он писал: «Надо отучиться от короткой фразы, она уместна только в моменты наиболее напряженного действия, быстрой смены жестов, настроений».

Синтаксис публицистических произведений, конечно, не может походить на синтаксис прозы Л.Н. Толстого или A.M. Горького. Но у А.П. Чехова есть чему поучиться. Выступая в качестве редактора, он рекомендовал по возможности упрощать сложные синтаксические конструкции. Так, редактируя рассказ В.Г. Короленко «Лес шумит», А.П. Чехов исключил при сокращении текста ряд придаточных предложений. Покажем эту правку (исключенные части предложений даны в скобках):

Усы у деда болтаются чуть не до пояса, глаза глядят тускло (точно дед все вспоминает что-то и не может припомнить). Дед наклонил голову и с минуту сидел в молчании (потом, когда он посмотрел на меня, в его глазах сквозь застилавшую их тусклую оболочку блеснула как будто искорка проснувшейся памяти). Вот придут скоро из лесу Максим и Захар, посмотри ты на них обоих: я ничего им не говорю, а только кто знал Романа и Опанаса, тому сразу видно, который на кого похож (хотя они уже тем людям не сыны, а внуки…). Вот же какие дела.

Конечно, правка-сокращение не сводится к бездумной «борьбе» с употреблением сложноподчиненных предложений, она обусловлена многими причинами эстетического характера и связана с общими задачами работы над текстом. Однако отказ редактора от придаточных частей, если они не несут важной информации, может быть обусловлен и стремлением к краткости и простоте. Для газетных материалов это особенно важно. Покажем выполненную редактором переработку сложных синтаксических конструкций в более простые на примере публицистического текста:

Неотредактированный текст:

Каждая эпоха, каждое событие накладывают свою печать на выразительность художественной формы архитектурных сооружений, которые являются каменной летописью города, можно даже сказать — каменной летописью мира, так как архитектура говорит о развитии культуры той или иной эпохи, напоминает об уровне культуры прошлого, не существующего уже народа.

Отредактированный текст

Каждая эпоха, каждое событие накладывают свою печать на выразительность художественной формы архитектурных сооружений. Они являются каменной летописью города, можно даже сказать — каменной летописью мира. Архитектура свидетельствует о развитии культуры в ту или иную эпоху, об уровне цивилизации уже не существующего народа.

Устранение стилистических недочетов и ошибок в предложениях при стилистической правке текста

Если выбор синтаксической конструкции не связан с проблемой сохранения авторского стиля, редактор вправе упорядочить структуру сложного предложения, уточнить союзную связь, сократить количество придаточных частей.

Однако в иных случаях стилистическая правка сводится к переработке простого предложения в сложное:

Неотредактированный текст

Нормальная, бесперебойная работа транспорта и безопасность движения пешеходов требует от школьников строгого подчинения сигналам светофора и обращения внимания на дорожные сигнальные знаки.

Отредактированный текст

Чтобы не мешать движению транспорта и обеспечить свою безопасность, школьники должны строго выполнять правила уличного движения: следить за сигналами светофора, обращать внимание на дорожные знаки.

Преобразование простого предложения в сложное позволило избавиться от частого повторения отглагольных существительных, нанизывания форм родительного падежа. Отредактированный вариант доступнее для восприятия, чем громоздкое простое предложение, неумело сконструированное автором.

Таким образом, в зависимости от контекста, оптимальным вариантом для выражения мысли могут быть и простые, и сложные предложения. Нужно только не загромождать синтаксические конструкции, строить фразу умело.

О низкой речевой культуре автора свидетельствуют ошибки в построении именно сложных предложений. Литературному редактору приходится исправлять беспорядочное расположение частей сложного предложения, искажающее грамматические связи и логическую последовательность развития мысли. Сравним два варианта предложения:

Неотредактированный текст

При неточном включении могут быть броски тока, превышающие 15-кратные от номинального тока генератора, который представляет опасность для механической целостности генератора, а также вызывают большие колебания с провалами напряжения.

Отредактированный текст

При неточном включении генератора возникают броски тока, превышающие в пятнадцать раз номинальный ток машины и вызывающие колебания напряжения и его провалы. Это может нарушить механическую целостность генератора.

В неотредактированном варианте определительная придаточная часть относится, конечно, не к слову генератор, как это может показаться из-за нарушения грамматических связей, а к существительному ток, хотя логичнее было бы ее связать со словом броски (тока). Заменив придаточную часть причастным оборотом, уточнившим грамматические и смысловые связи в предложении, и упорядочив расположение частей теперь уже простого предложения, редактор добился четкости в построении конструкции и правильности в выражении мысли.

Иногда причина нечеткости формулировки кроется в неправильном выборе средств связи частей сложного предложения — союзов, союзных сочетаний. Например, не сумел автор выразить причинную зависимость придаточной части в таком предложении: Название ресторана «Яр» было дано в соответствии с тем, что он находился на берегу речки Синички, которая в настоящее время, заключенная в трубы, протекает под землей.

Истинная причина необычного названия ресторана осталась загадкой, поскольку автор увлекся разъяснением несущественного для данного высказывания факта, осложнив конструкцию еще одной придаточной частью — определительной. Стилистическая правка требует переработки этого сложного предложения: Ресторан назвали «Яр», потому что он стоял над обрывом у крутого берега речки Синички (сейчас она заключена в трубы и протекает под землей).

В отредактированном предложении смысл прояснился благодаря четкому выражению причинной связи и выведению за скобки второстепенной информации о дальнейшей судьбе реки Синички.

Предложение строится из слов или словосочетаний, связанных друг с другом с помощью согласования, управления или примыкания. Эти типы подчинительной связи хорошо известны нам, однако в речи мы не всегда умеем ими пользоваться. Вспомните, как грибоедовский Скалозуб строит фразу: Мне совестно, как честный офицер. Следовало бы сказать: Мне, как честному офицеру, совестно, однако говорящий нарушил согласование членов предложения. Подобные ошибки называются смещением синтаксическо й конструкции : начало предложения дается в одном плане, а конец — в другом. У поэтов смещение конструкции иногда передает взволнованную речь, которая как бы не укладывается в строгие грамматические нормы. Например:

Когда с угрозами, и слезы на глазах,
Мой проклиная век, утраченный в пирах,
Она меня гнала… (A.C. Пушкин)

Однако в прозе смещение конструкции нас удивит. Так, в автобиографии В. Маяковского читаем: Я поэт. Этим и интересен. Об этом и пишу. Люблю ли я, или я азартный, о красотах кавказской природы также — только если это отстоялось словом. Это писал о себе человек, не боявшийся шокировать читателя и даже демонстрировавший свое пренебрежение к некоторым традициям… Нам это непозволительно. 

Смещение синтаксической конструкции — грубая ошибка, причиной ее является неправильное употребление словоформ. Например: Постоянное увлечение Андрея, поглощающее все его свободное время, — это спортом. В данном случае вместо ожидаемой формы именительного падежа существительного (его увлечение — спорт) появляется творительный, уместный в иной конструкции: Он увлекается спортом. Рассмотрим примеры стилистической правки предложений, в которых произошло смещение в сочетании словоформ:

Как видим, устранение этой синтаксической ошибки требует значительной переработки авторского текста: редактор может усложнить конструкцию, преобразовав простое предложение в сложное (второй пример), может заменить безличную конструкцию личной (третий пример) и т.д. Анализируя структуру предложения, редактор сталкивается и с незавершенностью синтаксических конструкций. Например: Только за последнее время принятые общероссийские законы об охране здоровья, укреплении семьи, землеустройстве, водопользовании, по усилению борьбы с преступностью.

Так, конечно, нельзя закончить предложение, но автор поставил точку и перешел к следующей фразе. Редактор должен восполнить отсутствие сказуемого в незавершенной конструкции, что можно сделать, почти не переделывая авторского текста, достаточно заменить полное причастие кратким: Только за последнее время приняты законы… Но редактор решил продолжить фразу: …принятые общероссийские законы об охране здоровья, укреплении семьи, землеустройстве, водопользовании, наконец, законы по усилению борьбы с преступностью свидетельствуют об укреплении правопорядка в нашем государстве.

Согласование подлежащего и сказуемого

К наиболее сложным случаям употребления синтаксических конструкций относится корреляция подлежащего и сказуемого. Например, как правильно: Несколько человек пришли или пришло? Сложности корреляции подлежащего и сказуемого возникают не только тогда, когда нужно правильно подобрать родовое окончание глаголов прошедшего времени (киви (птица) прилетела, а киви (плод) поспел), но и в том случае, если подлежащее выражено не одним словом, а так называемым количественно-именным сочетанием: Отец с сыном пришел или пришли? Шесть человек опоздало или опоздали? Два дерева упало или упали? Несколько друзей встретились или встретилось?

В некоторых случаях грамматическая форма зависит от значения: если отец привел сына, то обычно используют единственное число (пришел), а если они пришли вместе («на равных»), то употребляется множественное число (пришли). При выборе сказуемого в трех других примерах действует следующее правило: если подлежащее одушевленное или указывает на лицо, то чаще употребляется множественное число (Пять/несколько девочек вернулись раньше); если подлежащее относится к неодушевленным существительным, то сказуемое обычно приобретает форму среднего рода (Пять/несколько яблок поспело).


  1. https://cyberleninka.ru/article/n/spetsifika-narusheniya-otdelnyh-sintaksicheskih-norm-sovremennogo-russkogo-yazyka
  2. Голуб И.Б. Литературное редактирование: учеб. пособие / И.Б. Голуб. — М.: Логос, 2010. — 432 с.

27.10.2019, 8808 просмотров.

Практическое
занятие по дисциплине «Русский язык»

Тема:
Анализ ошибок и недочетов в построении сложного предложения.

Цель занятия: способствовать развитию умения анализировать  ошибоки и недочеты в
построении сложных предложений; развивать речемыслительную деятельность
обучающихся,
навыки правильной постановки знаков препинания в сложноподчинённых предложениях
посредством выполнения тренировочных упражнений; формировать устойчивую мотивацию
к изучению русского языка, к достижению пунктуационной грамотности —  как
важного качества будущего специалиста.

Результаты:

Л(1,3,6)

М(1,2,4,5,6)

П(1,2,3,4)

Норма времени: 90
минут.

Оснащение рабочего
места
:
инструкционная карта.

Контрольные
вопросы при допуске
:

1.Назовите
отличительные черты простого предложения от сложного.

2.Какие
виды союзных сложных предложений вы знаете?

3.Чем
отличается сложносочиненное от сложноподчиненного предложения?

4.
Назовите основные ошибки в построении сложных предложений.

Теоретическая информация

Среди ошибок
и недочётов
, связанных с употреблением
сложных предложений
наиболее распространёнными
являются следующие:

 — 
неправильное построение самой конструкции предложения;

— 
использование излишне громоздких конструкций.

1. Одним
из самых распространенных недочётов является загромождение сложного
предложения придаточными предложениями.

Ср.: Заявление
представителей иностранных кругов, игнорирующее тот факт, что торговые
отношения, которые в последние годы неуклонно развивались и проявляют тенденцию
к дальнейшему росту, свидетельствует о том, что кто-то по-прежнему
заинтересован в сохранении атмосферы «холодной войны» и ликвидации массового
стремления к дружбе, которое охватило народы Европы и Америки, а это не может
не отразиться на действиях нашего государства, которое продолжает рассчитывать
на успех переговоров, хотя понимает, что достичь прогресса в таких переговорах
будет нелегко, но мы привыкли преодолевать трудности.

►Неуместно
также нанизывание однотипных синтаксических конструкций сложносочинённого или
сложноподчинённого предложения.

Ср.: Кай
старался отцепить санки, а они точно приросли, а седок поворачивался и мотал
головой, а Герда только глядела вслед другу.

2. В
ряде случаев одна и та же ситуация может быть выражена с помощью как
сложносочинённых, так и сложноподчинённых предложений.

Ср.: Он
вошёл, 
и мы
встали; 
Когда он
вошёл, мы встали.

►В то
же время в речи часто наблюдаются случаи «сбоя структуры»: предложение, начатое
как сложноподчинённое, заканчивается как сложносочинённое, и наоборот. Это
недопустимо!

Ср.: Когда Мурке
надоедало возиться с котятами, 
и она
уходила куда-нибудь поспать.

3. К
грамматическим ошибкам относится также использование разнотипных частей
сложного предложения.

Например: Представитель
нашей делегации выдвинул два положения: 1) без России Совет Европы не может
полноценно функционировать; 2) необходимость стабилизации положения на Кавказе.
 В
данном бессоюзном предложении при пояснении в первом случае используется
двусоставное предложение (Совет не может функционировать, во втором –
назывное предложение с одним главным членом – подлежащим необходимость (эта
часть может быть охарактеризована и в качестве словосочетания, а не
предложения).

4. Речевым
недочётом при построении сложного предложения является так называемое смещение
конструкции.

Например: Последнее,
о чём я скажу, это о перерасходе средств.
 В данном случае в главном
предложении (Последнее… это о перерасходе средств) произошло смещение
конструкции под влиянием придаточного предложения (о чём я скажу).
Грамматически более корректной была бы конструкция: Последнее, о чём я
скажу, это перерасход средств
.

5. Очень
распространены в речи ошибки и недочёты, связанные с неправильным
употреблением союзов и союзных слов:

а) использование
одновременно нескольких союзов, союзных слов, лишней частицы 
бы при
союзе 
чтобы.

Например:

1. Мать
заболела, 
но тем не менее, однако, она
не осталась дома.
 Корректная конструкция: Мать
заболела, 
но тем не менеене
осталась дома.

2. Он
подумал, 
что кончена ли жизнь
в тридцать лет.
 Корректная конструкция: Он
подумал, кончена 
ли жизнь
в тридцать лет.

3. Надо, чтобы он
зашёл 
бы ко
мне.
 Корректная конструкция: Надо, чтобы он зашёл ко мне;

б) неуместная
вставка или, напротив, неправомерное опущение указательных слов: 
тот,
то, такой
 и др.

Например:

1. Ваша
ошибка заключается, что вы слишком торопитесь опубликовать свои стих
 –
опущено указательное слово. Корректная конструкция: заключается в 
том,
что…

2. Необходимо то,
чтобы ты зашёл ко мне
 – указательное
слово то ошибочно употребляется при наречии, не требующем
такого распространения;

в) неуместное
использование одного союза или союзного слова вместо другого:

например:

Нельзя
мириться 
с таким положением, когда работы
выполняются наспех
 – следовало бы: Нельзя
мириться 
с тем, чтоработы
выполняются наспех
;

г) неуместное
использование одновременно союза и союзного слова при однородном подчинении:

например:

Писатель
понял, 
что Булька
бежал за ним 20 вёрст и 
какой преданный
друг у него есть
 – следовало бы: Писатель
понял, 
чтоБулька,
бежавший за ним 20 вёрст, – преданный друг; Писатель понял, что Булька бежал за
ним 20 вёрст и что пёс – преданный друг.

6. В
некоторых типах сложноподчинённых предложений могут использоваться инфинитивные
предложения (сказуемое выражено в них независимым инфинитивом). Это возможно
только в том случае, если в главном и в придаточном предложениях говорится об
одном и том же действующем лице:

Чтобы
не опоздать на поезд, мне пришлось взять такси.

►Нельзя
использовать инфинитивные конструкции в тех случаях, когда в главном и в
придаточном предложениях действуют разные субъекты.

Например: Чтобы
написать сочинение, учительница рассказала нам о плане
. В данном случае
сочинение будут писать школьники, а о плане им рассказала учительница! Поэтому
грамматически корректной будет конструкция: Чтобы мы смогли написать
сочинение, учительница рассказала нам о плане
.

►Не
рекомендуется также использовать целевые придаточные с союзом 
чтобы (особенно
в позиции – после главного предложения!) в том случае, если главное предложение
выражает не активное действие, а пассивное состояние, отношение и т.п.:

Лампочки
на катке 
горят ровно,
чтобы освещать каток.

7. Невыраженность
существенного звена мысли, связанного с восприятием какого-либо явления или его
оценкой, часто приводит к речевым недочётам, которые выглядят как логические
ошибки:

В
библиотеке холодно, потому что на стул надета шаль.

8. При использовании определительных придаточных
предложений типичными являются следующие ошибки и недочеты:

а) придаточные предложения отрываются от
определяемого слова:

Холодный дождик смочил поля, в котором так
нуждались посевы
;

б) союзные слова ошибочно согласуются не с тем
существительным, которое является определяемым:

Кролик – зверь, который живёт в лесу;

в) союзное слово который неправомерно
переносится в середину придаточного предложения:

Гоги – это грузинский мальчик, в детстве
который был взят в плен русским спецназом
;

г) неправомерно опускается определяемое слово
в главном предложении.

например:

 
Командир пригласил, кто спас девочку.
 
пояснение:
В данном случае придаточное предложение должно
соотноситься либо с существительным, либо с местоимением:
 
Командир пригласил человека, который спас
девочку;
 
Командир пригласил того, кто спас девочку

Ход
занятия

Задание 1. Произведите правку текста, устранив имеющиеся
грамматические ошибки и речевые недочеты (выделены курсивом).

1. Представив такие сюжеты описания, появляется какое-то вдохновение и,
естественно, хочется увидеть такое же чудо, этот прекрасный мир,
описываемый в данном тексте.

2. Созерцая родную природу, память его запечатлела множество
удивительного и прекрасного.

3. Одной из главных проблем, поставленной
автором, 
является то, что для писателя очень важно мнение читателя о написанном
произведении или простом стихотворении.

4. С автором я согласен, ведь стихотворение должно
быть написано 
на чувствах поэта.

5. Бывает, что и не поняв смысл стихотворения, на нас находят различные
эмоции, 
мы все равно улавливаем настроение, 
характерное данной поэзии.

6. Все мелочи мастерства поэта, все тонкости, они у
всех поэтов разные, у каждого поэта свой взгляд на мир, 
свои мысли, свой жанр в
поэзии.

7. Копирование людей заложено в нас еще
с самого детства. В детстве мы хотим быть такими же, как наши
родители или старшие товарищи. Со временем как мы взрослеем, хотим
походить на идеалов и копируем за ними жесты,
разговоры.

8. Учитель не только делится со своими знаниями, но и с
жизненным опытом.

9. Авторская позиция всем ясна и понятна. Когда страну покидают те, на
кого была возложена большая надежда на то, что они выведут страну на новый
уровень, 
сделаввеличайшие открытия и изобретения в своей сфере
деятельности, 
то всегда становится грустно на душе и обидно за
соотечественников.

10. Все ученые, великие писатели, художники, никто из
них не был признан 
в момент их жизни, зато сейчас мы
пользуемся их открытиями, любуемся картинами, читаем книги.

Задание 2. Установите, есть ли в приведенных фрагментах ошибка
«нарушение границ предложения». Внесите необходимые исправления.

1. Конечно, рецепта от бездуховности нет, но мы
должны понять ее истоки, объяснить людям, с чем они имеют дело. И попытаться
сделать так, чтобы наши близкие, друзья, родные, да и просто незнакомые люди
всегда были окружены вниманием и заботой.

2. Автор рассказывает нам о драматическом эпизоде
из жизни Михаила Богдановича Барклая-де-Толли. Триумф которого так и не
наступил. В.Лаптев хочет, чтобы читатель задумался о том, как живется гениям.

3. Автор хотел показать, насколько хуже становится
молодежь с каждым годом. Насколько часто происходят конфликты в семье.

4. Мы с вами в своей жизни встречаемся с разными
проблемами. Которые не в силах все решить.

5. Самым ярким примером из обыденной жизни может
служить обыкновенная семья, в которой есть дети. Ведь каждый родитель не раз
упрекнул за поведение, которое не соответствует его уже устоявшимся моральным
нормам, но у каждого поколения они свои, и именно это служит причиной
конфликтов между поколениями.

6. Позиция автора заключается в том, что человек,
не имевший учительского диплома, мог так заинтересовать детей своими
рассказами. Повидавшая на своем жизненном пути много интересного – заведующая
библиотекой.

7. Автор текста ищет в биографиях великих ученых
примеры «порядочности, великодушия, добра, чести, красоты души». Вот были же
времена, когда честь была выше всего. Когда важнее правое дело, чем своя
дальнейшая жизнь.

8. Нужно преодолевать и контролировать ревность, в
этом и заключается любовь, в преодолении различных трудностей, встречающихся у
нее на пути. И не боясь переступить через свою гордость, через свои амбиции
ради своего возлюбленного. И это не значит уничтожить свое «я», это значит
обрести те чувства, которые дарованы человеку.

9. В тексте А.Яблокова рассказывается о
недовольстве людей старшего поколения нынешней молодежью. Их легкомыслием и
несерьезностью. Говорится, что молодежь безнадежно испорчена пороками.

10. Судьбы многих гениальных людей были печальными.
За свое же творчество, за свои заслуги они пострадали. Они были непризнанными.

Задание 3. Установите, какие ошибки допущены в данных предложениях,
классифицируйте их. Предложите разные способы исправления.

1. Те, кто знали Левитана близко, говорили, что он
любил русскую природу фанатически, почти исступленно.

2. Художник заставляет нас остановиться около
старого могучего дуба, возле трогательной тонкой осинки с начинающими краснеть
листочками и обратить внимание, войти в пронизанную светом березовую рощу и в
темную зелень высоких сосен.

3. О Левитане говорили не только как о художнике
исключительного дарования, а как о человеке чуткой души, тонкого восприятия
жизни.

4. Будучи в Италии, Левитана радовало все:
итальянская весна, ее глубокое синее небо, изумрудная трава, цветущий миндаль.

5. Наиболее характерные для полотен художника
летние дни, горячее солнце, сочная зелень деревьев.

6. Картина «Явление Христа народу», поглотившая всю
творческую энергию А.Иванова, разочаровала его и не закончена.

7. В эскизах художник намечает композицию и колорит
картины: отбирает и располагает предметы и действующие лица, распределяет
краски, свет и тени.

8. В картине «Сватовство майора» Федотов показал
как нрав русского купечества, так и дворянства сороковых годов XIX века.

9. Как вы относитесь к оценке серовского портрета
«Иды Рубинштейн» Репина?

10. На всех своих картинах Репин размышляет об
основных проблемах своего времени.

Задание 4. Прочитайте фрагменты ученических работ, разберитесь в их
содержании. Предложите свои варианты правки.

1. Известно также много случаев про исполнителей
песен нашей эстрады, они пытаются исполнить те старые песни, которые еще
слушали наши деды, хотя ведь можно сочинить что-то более интересное со своими
возможностями.

2. Молодежь в наши дни, можно сказать, не
увлекается искусством. Большинство из них считают это глупым, немодным, пустой
тратой времени. Хотя, может быть, они просто не улавливают мысль поэта?

3. Я могу сказать про себя, что я любитель поэзии.
Я люблю читать стихи и даже иногда пишу их сама. И могу сказать, что и мне
попадались много стихотворений с непонятным смыслом на первый взгляд, но если
прочитать еще раз и призадуматься, начинаешь понимать смысл.

4. Я полностью поддерживаю точку зрения автора в
том, что не понятое однажды позже может оказаться золотым источником, и то, что
перечитывание стихотворения усиливает наше эмоциональное отношение к нему. И
только настоящие ценители искусства не дают «пропасть» настоящей поэзии. Но им
может стать даже самый простой человек, надо только стремиться к постижению все
новых и новых тайн.

5. На мой взгляд, в любой школе должно работать
достаточное количество учителей-мужчин, которые бы могли своей строгостью и
сдержанностью дополнять такие качества, как мягкость и безмерную любовь к
детям, присущие почти всем учителям-женщинам.

6. Книга – это наша жизнь и история. Главным ее
противником на сегодняшний день, по мнению автора, является компьютер и
Интернет.

7. Одной из проблем, поднятых автором данного
текста, является проблема духовности и бездуховности человека, т.е. влияние
этих качеств на самих людей. При отсутствии духовности можно еще сильнее
обидеть человека, лезя к нему со своими глупыми, бестактными вопросами. И,
наоборот, проявил духовность к другому человеку, войдя в его положение, став
как бы его частицей, можно помочь ему преодолеть трудности.

8. Проблема данного текста состоит в том, что нам
важно только то, что мы будем делать дальше, а не то, что случится с окружающим
миром. Я считаю, что написанный текст Трифоновым очень злободневен и от этого
страдает вся планета.

9. Главным воспоминанием из детства для
В.Амлинского стало времяпровождение в библиотеке. В то послевоенное время дети
очень любили читать, у них не было никаких телевизоров и компьютеров, как у
нынешних детей. С детства родители их приучали к книгам, а не как сейчас – садят
смотреть мультфильмы. Тогда дети собирались в библиотеке, слушали рассказы,
делились эмоциями, их выслушивали взрослые люди. Автор вспоминает заведующую
этой библиотеки, поистине душевного человека. Таких людей сейчас очень мало –
таких, которые помогут, выслушают, посоветуют. Еще к таким людям относятся
писатели, которые приходили к детям со своими историями, рассказами. В общем,
дети не сидели на месте, а проводили время с толком.

10. Автор пишет о людях, которые вели себя поистине
мужественно, честно, благородно. Эти люди не думают о своем благополучии. Они
выступают за правду, за справедливость. От этого им становится хорошо, спокойно
на душе. Все их поступки направлены на улучшение жизни других, на защиту их
прав и свобод. При этом они не задумываются о том, что тем самым могут
усложнить свою жизнь. Это их не пугает.

Задание 5.Выполните тестовое задание.Укажите предложение с
грамматической ошибкой (с нарушением синтаксической нормы). Ответ дайте по
образцу: 1 – 3), 2 – 4)…

1.

 1) Мой соперник выиграл партию в шахматы благодаря
тонкого понимания сложной позиции.

2) Студенты проходили практику в одном из цехов
завода, недавно реконструированном.

3) В.Стасов, который был известен как музыкальный
критик, уделял в своей публицистике много внимания и изобразительному
искусству.

4) Одновременно с работой над «Явлением Христа
народу» Иванов создавал эскизы на библейские сюжеты и рисунки-акварели
«Сотворение мира».

2. 

1) По окончании переговоров между странами
установились дипломатические отношения.

2) Мы должны радоваться жизни и ценить, ведь она у
нас одна и пережить ее заново возможности уже не будет.

3) В.Г. Перов создал рисунок «Похороны Гоголя
героями его произведений», в котором на небольшом художественном пространстве
собрал в одну группу основных персонажей писателя.

4) Туманы в Лондоне бывают если не каждый день, то
через день непременно.

3. 

1) Картина И.Е. Репина «Бурлаки на Волге»
была показана на всемирных выставках 1873 и 1878 годов и имела большой успех.

2) Воспитанники Академии художеств очень любили и
уважали своего учителя Павла Петровича Чистякова.

3) Идея всемирного братства людей, золотого века,
всеобщего счастья – самые дорогие мечты писателя с юношеских лет и до конца его
дней.

4) По прошествии многих лет с момента создания
произведения искусства могут оцениваться иначе.

4. 

1) Многочисленные факты, накопленные наукой,
подтвердили правильность гипотезы, выдвинутой молодым ученым.

2) Мужество и отчаяние, скорбь, тоску и одиночество
– все эти чувства выражают глаза матери в эскизе М.А. Врубеля.

3) Главные темы «Униженных и оскорбленных»,
«Записок из Мертвого дома», «Записок из подполья» – тема бунта и тема
героя-индивидуалиста – синтезировались потом в «Преступлении и наказании».

4) Открытие памятника Пушкина в Москве в июне 1880
года было незаурядное событие в истории русской культуры.

5. 

1) Мой подопечный был не столько расстроен, сколько
удивлен сложившейся ситуацией.

2) Картина В.Г. Перова «Проводы покойника»
проникнута чувством боли, сострадания, настроением безысходной тоски.

3) Те, кто не набрал нужного количества баллов,
будут вынуждены выполнить зачетную работу еще раз.

4) На мой взгляд, в любой школе должно работать
достаточное количество учителей-мужчин, которые бы могли своей строгостью и
сдержанностью дополнить такие качества, как мягкость и безмерную любовь к
детям, присущие почти всем учителям-женщинам.

6. 

1) Талант Паустовского фиксирует наше внимание на
прекрасном.

2) Тихон Кабанов, муж Катерины, был человек
безвольный, полностью подчинившимся матери.

3) В 1859 году «исповедь-роман» Достоевского
«Записки из Мертвого дома» еще не был начат.

4) Зарево распространилось не только над центром
города, но и далеко вокруг.

7. 

1) Все передовое, что было в русской литературе
60-х годов XIX века, сосредоточивалось в журнале «Современник».

2) Истоки «Преступления и наказания» восходят ко
времени каторги.

3) Взрослея, все лучше начинаешь понимать своего
учителя; так же, как и прежде, восхищаешься и стремишься быть похожим на него.

4) Нам порой кажется, что мы не только знаем все,
но нас ничем не удивишь.

8. 

1) Речь Достоевского сделала открытие памятника
Пушкину в Москве выдающимся, грандиозным событием как в истории русской, так и
мировой культуры.

2) В общество человек может принести с собой
атмосферу подозрительности, какого-то тягостного молчания, а может внести сразу
радость, свет.

3) Для «Прогулки короля» А.Н. Бенуа выбирает
таинственное время года – осень – и самое таинственное время суток – вечер.

4) Вами своевременно оплачен проезд в маршрутном
такси?

9.

 1) Живая крыса, о которой рассказывает Ю.Трифонов,
должна быть разрезана, чтобы учиться анатомии, в так называемой «пригодинской»
школе.

2) Мама не то что сердилась, но все-таки была
недовольна.

3) Пришлось с трудом пробираться сквозь заросли,
постоянно заслонявшие нам дорогу.

4) Железнодорожный состав отправляется с третьего
пути согласно расписанию.

10. 

1) Никто больше не нарушал тишину протоков и рек,
не обрывал блесной холодные речные лилии и не восторгался вслух тем, чем лучше
всего восторгаться без слов.

2) Достоевский принадлежит к тем писателям,
биография которого тесно связана с творчеством.

3) Благодаря установившейся ясной погоде урожай был
собран вовремя.

4) Все, кто интересуется художественной жизнью
России конца XIX – начала XX века, слышали о творческом объединении молодых
художников, известном под названием «Мир искусства».

Ключ:

1 – 1); 2 – 2); 3 – 3); 4 – 4); 5 – 4); 6 – 2); 7 –
3); 8 – 1); 9 – 1); 10 – 2).

Задание
6.

Прочитайте. Найдите ошибки и недочёты в построении сложноподчинённых
предложений. Спишите, исправляя данные предложения.

I.
1) Книга рассказывает нам интересную историю жизни людей, которую подарил мне
товарищ. 2) С горы были видны луга и густые хвойные леса, которые пестрели
цветами. 3) Туманные полосы начали понемногу алеть и расходиться, в которых
тонули луга и пашни. 4) В штабе дивизии получили известие, что река вскрылась,
что переправа ещё не налажена. 5) На другой день Петя пошёл к своему товарищу Оболенскому,
которому было пятнадцать лет, который тоже поступил в полк. 6) Альпинисты
подошли к лагерю, где был назначен сбор всех отрядов, откуда должно было
начаться восхождение на Эльбрус. 7) Туристы свернули в лес, который тянулся до
реки, по которой ходили теплоходы. 8) Посёлок, возле которого отряд
расположился на отдых, находился на опушке леса, который славился обилием
грибов. 9) Разведчики тихо подползли к реке, спуск к которой был покрыт густой
и высокой травой, и решили перебраться на другой берег, который зарос камышом.

II.
1) Главное, на что обращает внимание автор, — это на тщательный отбор языковых
средств. 2) Зритель сочувствует жене Клеща Анне, всю жизнь видевшей лишь побои,
обиды и которая в свои тридцать лет больна чахоткой. 3) Авторские ремарки
указывают на разные детали, например на стук топора, который доносится из-за
кулис. 4) Когда его спросили, почему он убил девушку, Ларра ответил, что зачем
я буду объяснять вам свои поступки. 5) Тренер сказал нам, чтобы мы передали
пловцам, чтобы они пришли в бассейн в субботу вечером.

Задание 7. Охарактеризуйте ошибки в
сложных предложениях. Внесите стилистическую правку.

1) В минувшую субботу для родителей, у кого дети-дошкольники, в
Доме культуры состоялась конференция. 2) Докладчик заострил внимание на
вопросе, что нередко поднимается на подобных собраниях. 3) Выступающий, который
сообщил эти данные и оказавшийся страстным патриотом речного транспорта, привел
интересное сравнение. 4) Автор сгоряча заметил рецензенту, что как же вы можете
не замечать того нового, что имеется в книге. 5) Основное, на что следует
обратить внимание, — это на нарушение дисциплины. 6. Кто нуждается в
дополнительных занятиях, необходимо тому обеспечить их. 7) В сборник вошли
частушки, сложенные не только на Смоленщине, но и в других областях, которые
бытуют в Смоленской губернии.

Задание 8. Найдите ошибки и недочёты в
употреблении сложных предложений. Определите тип ошибки и исправьте
предложения.

1) Врачи считают, что состояние больного настолько ухудшилось, что
вполне вероятно, что в течение нескольких часов может наступить смерть. 2)
Газета называет эти мероприятия горькой пилюлей для сторонников «холодной
войны», которая должна быть проглочена. 3) Кто написал отчёт о педагогической
практике, нужно сдать его руководителю. 4) На заводе есть 20 автоматов для
обработки шурупов, которые для работы на них требуют специальных
приспособлений. 5) М.Ю. Лермонтов пишет в своём стихотворении, что «и ненавидим
мы, и любим мы случайно». 6)  Подул сильный ветер, но тем не менее,
однако, дождь не прекратился. 7) В своей отповеди Онегин говорит Татьяне, что
«я вас люблю любовью брата». 8) Каждый раз задаёшь себе вопрос, что может ли
быть две правды. 9) Соня долго размышляла, что будет ли преступлением погубить
свою душу, чтобы спасти бы жизнь детей. 10) Островский показал силу любви
Ларисы и что с ней стало из-за золота. 11) Искали его три дня, но всё напрасно,
но потом нашли в степи без чувства. 12) Как только Троекуров велел позвать
Шабашкина, и через минуту заседатель стоял перед барином. 13) Когда приходишь
на бульвар, и смотришь, как тихо кружатся листья, и на душе становится радостно
и тепло. 14) Он подал прошение генералу, чтобы дать ему роту солдат для
освобождения Белогорской крепости. 15) Когда я заглянул в колодец, то он был
очень глубокий. 16) Собака гуляла с тётей Варей, на которую был надет
намордник. 17) К светскому обществу князь Андрей относится с презрением, в
котором принуждён бывать. 18) Пьеса «На дне» – это произведение, в которой
содержится обвинение капитализму. 19) Он видит парк из тонких чахлых деревьев,
в газете о котором писалось, что это великолепный сад. 20) Борис, дядя которого
решает послать в Сибирь, приходит проститься с Катериной. 21) Писатель понял,
что какой преданный друг у него есть. 22) Тарас сказал сыновьям, что завтра же
едем в Запорожье. 23) Гринёв любит Машу от чистого сердца, а Швабрин любит,
чтобы замучить. 24) На стене висит портрет, когда он был молодой. 25) Это
подтверждают слова Тихона, когда он видит труп Катерины. 26) Воробьёв, которые
осторожно подлетают, особенно удивляют часы, которые стоят на врытом в землю
столике. 27) Мцыри узнал о себе, что он не боится лишений, какой он смелый,
какой обладает несокрушимой волей. 28) Подбородок у меня закруглённый, после
которого идёт тонкая шея, которую каждый раз натирает белая рубашка.

Требования к
отчету:
задания
должны быть выполнены в тетради для практических работ.

Критерии оценки:

оценка «5» – верно
выполнено 7-8 заданий;

оценка «4» – верно
выполнено 5-6заданий;

оценка «3» – верно
выполнено 3-4 задания;

оценка «2» – верно
выполнено менее 3-х заданий.

Список литературы

1.Воителева Т.М. Русский
язык и литература. Русский язык (базовый уровень): учебник для 11 класса
общеобразовательной школы. — М., 2014.

2.Воителева Т.М. Русский
язык: сб. упражнений: учеб, пособие сред.проф. образования. — М., 2014.

3.Гольцова Н.Г., Шамшин И.В., Мищерина М.А. Русский
язык и литература. Русский язык (базовый уровень). 10—11 классы:  2 ч. — М.,
2014.

4. Интернет-ресурс: инфоурок.ру.

Результаты:

личностные

1.Воспитание
уважения к русскому (родному) языку, который сохраняет и отражает культурные
и нравственные ценности, накопленные народом на протяжении веков, осознание
связи языка и истории, культуры русского и других народов;

2.Понимание
роли родного языка как основы успешной социализации личности;

3.Осознание
эстетической ценности, потребности сохранить чистоту русского языка как
явления национальной культуры;

4.Формирование
мировоззрения, соответствующего современному уровню развития науки и
общественной практики, основанного на диалоге культур, а также различных форм
общественного сознания, осознание своего места в поликультурном мире;

5.Способность
к речевому самоконтролю; оцениванию устных и письменных высказываний с точки
зрения языкового оформления, эффективности достижения поставленных
коммуникативных задач;

6.Готовность
и способность к самостоятельной, творческой и ответственной деятельности;

7.Способность
к самооценке на основе наблюдения за собственной речью, потребность речевого
самосовершенствования;

метапредметные

1.Владение
всеми видами речевой деятельности: аудированием, чтением (пониманием),
говорением, письмом;

2.Владение
языковыми средствами — умение ясно, логично и точно излагать свою точку
зрения, использовать адекватные языковые средства; использование
приобретенных знаний и умений для анализа языковых явлений на межпредметном
уровне;

3.Применение
навыков сотрудничества со сверстниками, детьми младшего возраста, взрослыми в
процессе речевого общения, образовательной, общественно полезной,
учебно-исследовательской, проектной и других видах деятельности;

4.Овладение
нормами речевого поведения в различных ситуациях межличностного и
межкультурного общения;

5.Готовность
и способность к самостоятельной информационно-познавательной деятельности,
включая умение ориентироваться в различных источниках информации, критически
оценивать и интерпретировать информацию, получаемую из различных источников;

6.Умение
извлекать необходимую информацию из различных источников: учебно-научных
текстов, справочной литературы, средств массовой информации, информационных и
коммуникационных технологий для решения когнитивных, коммуникативных и
организационных задач в процессе изучения русского языка;

предметные

1.Сформированность
понятий о нормах русского литературного языка и применение знаний о них в
речевой практике;

2.Сформированность
умений создавать устные и письменные монологические и диалогические
высказывания различных типов и жанров в учебно-научной (на материале
изучаемых учебных дисциплин), социально-культурной и деловой сферах общения;

3.Владение
навыками самоанализа и самооценки на основе наблюдений за собственной речью;

4.Владение
умением анализировать текст с точки зрения наличия в нем явной и скрытой,
основной и второстепенной информации;

5.Владение
умением представлять тексты в виде тезисов, конспектов, аннотаций, рефератов,
сочинений различных жанров;

6.Сформированность
представлений об изобразительно-выразительных возможностях русского языка;

7.Сформированность
умений учитывать исторический, историко-культурный контекст и контекст творчества
писателя в процессе анализа текста;

8.Способность
выявлять в художественных текстах образы, темы и проблемы и выражать свое
отношение к теме, проблеме текста в развернутых аргументированных устных и
письменных высказываниях;

9.Владение
навыками анализа текста с учетом их стилистической и жанровородовой
специфики; осознание художественной картины жизни, созданной в литературном
произведении, в единстве эмоционального личностного восприятия и
интеллектуального понимания;

10.Сформированность
представлений о системе стилей языка художественной литературы.

Вместо точек вставьте слова, которые наиболее точно выражают мысль; мотивируйте свой выбор. (Слова, которые использовал автор, см. в конце упражнения.) Человек … (изобрел, нашел, отыскал, придумал, создал) слова для всего, что обнаружено им … (в мире, во вселенной, на земле). Но этого мало. Он … (назвал, объяснил, определил, указал на) всякое действие и состояние. Он … (назвал, обозначил, объяснил, окрестил,определил) словами свойства и качества всего, что его окружает. Словарь … (воспроизводит, определяет, отображает, отражает, фиксирует) все изменения, … (происходящие, совершающиеся, существующие) в мире. Он … (запечатлел, отразил, сохранил) опыт и мудрость веков и, не отставая, сопутствует жизни, … (движению, прогрессу, развитию) техники, науки, искусства. Он может … (выделить, назвать, обозначить, определить, указать на) любую вещь и располагает средствами для … (выражения, обозначения, объяснения, передачи, сообщения) самых отвлеченных и обобщенных идей и понятий. (По С. Маршаку.) Примечание. Нашел, во вселенной, назвал, определил, отражает, происходящие, запечатлел, развитию, назвать, выражения.

Памятка эксперту «Классификация ошибок»

К 7. Орфографические ошибки

Орфографические ошибки- неправильное написание слова, допущенное на письме, обычно в слабой фонетической позиции.

Однотипные и неоднотипные орфографические ошибки:

  • Однотипные- ошибки на одно и то же орфографическое правило. Эти ошибки исправляются, но при подсчете общего количества допущенных ошибок считаются 3 за 1, каждая последующая считается самостоятельной ошибкой. Примечание: не считаются однотипными ошибки, допущенные в словах с проверяемыми гласными в корне слова.

  • Повторяющиеся- ошибки в одном и том же повторяющемся слове или в корне однокоренных слов. Эти ошибки также считаются за одну.

Грубые и негрубые орфографические ошибки:

К негрубым орфографическим ошибкам относятся:

  • исключения из правил;

  • строчная и прописная буквы в собственных наименованиях;

  • не регулируемые правилами слитные и раздельные написания наречий;

  • слитное и раздельное написание НЕ с прилагательными и причастиями, выступающими в роли сказуемых;

  • различение НЕ и Ни в оборотах никто иной не…, ничто иное не…, не кто иной, как…, не что иное, как…, а также во фразах типа: Куда только он не обращался! Куда только он ни обращался, никто не мог дать ему ответ;

  • написание собственных имен нерусского происхождения;

  • написание И и Ы после приставок.

Другие виды ошибок:

  • Графические ошибки ( пи проверке не учитываются)- разновидность ошибок, связанных с графикой ( различные приемы сокращения слов, использование пробелов между словами, различные подчеркивания, шрифтовые выделения).

  • Описки-ошибки, вызванные невнимательностью пишущего или поспешностью написания. Они исправляются, но не выносятся на поля и не учитываются при подсчете ошибок. К опискам относятся:

  • пропуски букв;

  • перестановка букв;

  • замена одних букв другими;

  • добавление лишних букв.

К 8. Пунктуационные ошибки

Пунктуационные ошибки-ошибки, связанные с нарушением пунктуационных правил. 2 негрубые пунктуационные ошибки считаются за 1.

К негрубым пунктуационным ошибкам относятся :

  • употребление одного знака препинания вместо другого;

  • пропуск одного из двойных знаков препинания;

  • нарушение последовательности сочетающихся знаков или пропуск одного из сочетающихся знаков;

  • не относится к ошибкам авторская пунктуация.

Понятие об однотипных ошибках НЕ распространяется на пунктуационные ошибки.

К 9. Классификация грамматических ошибок

  • ошибки в образовании слов, связанных с нарушением словообразовательных форм;

  • ошибки в образовании форм различных частей речи, связанные с нарушением морфологических норм;

  • ошибки в построении словосочетаний, связанные с нарушением синтаксических норм;

  • ошибки в построении простого предложения, связанные с нарушением синтаксических норм;

  • ошибки в построении сложного предложения, связанные с нарушением синтаксических норм.

  1. Ошибки в образовании отдельных слов и форм различных частей речи

1.

Неправильное использование приставок: зафотографировали (вм. сфотографировали).

2.

Неправильное использование суффиксов: одеяло мокнуло ( вм. мокло).

3.

Ошибки при образовании сложных слов: полуумный (вм. полоумный).

4.

Неправильное образование формы И.п. мн.ч.: торта (вм. торты), договора (вм. договоры).

5.

Неправильная форма рода: лёгкая тюль (вм. лёгкий ).

6.

Неправильная форма числа: вышел на подмосток ( вм. на подмостки).

7.

Нарушение норм склонения иноязычных фамилий: Повесть Аркадия и Георгия Вайнер

( вм. Вайнеров).

8.

Неправильное образование формы Р.п. мн.ч.: Пять килограмм (вм. пять килограммов),

много грузинов (вм. грузин), новыми дверями (вм. дверьми).

9.

Ошибки в категории одушевленных и неодушевленных существительных:

Я забрал его ферзь (вм. ферьзя).

10.

Склонение несклоняемого существительного: Мы побывали на горном плате (вм. плато).

11.

Неверное образование форм кратких прилагательных:

Вывод беспочвенен (лучше беспочвен), ему свойственен оптимизм ( вм. свойствен).

12.

Ошибки при образовании степеней сравнения: Самый талантливейший поэт

( вм. самый талантливый), более тяжелее (вм. более тяжело).

13.

Неправильное образование местоимений 3-го лица мн.ч.: ихний сын (вм. их).

14.

Ошибки в образовании личных форм глаголов: ветер колыхает листву ( вм. колышет).

15

Ошибки в образовании повелительного наклонения: едь (вм. езжай), ляжь (вм. ляг).

16.

Неверное образование видовых форм: институт организовывает подготовительные курсы (вм. организует).

17.

Неправильное образование действительных и страдательных причастий:

Изделия, выпускающиеся в России (вм. выпускаемые).

18.

Ошибки при образовании деепричастий: Пиша диктант, нужно быть внимательным (нет литературной формы деепричастия).

19.

Ошибки при образовании наречий ( использование разговорной формы вместо литературной): тута, здеся (вм. тут, здесь).

  1. Ошибки в согласовании

1.

Неправильное использование сказуемого при подлежащем, выраженном собирательными существительными: Стая лебедей летели (вм. летела). Большинство участников собрания проголосовали ( вм. проголосовало).

2.

Неправильное употребление числа сказуемого: Там жила бабушка и ее внук (вм. жили).

3.

Согласование обобщающего слова и однородных членов: На конференции выступили делегаты из следующих городов: Красноярск, Москва… (вм. Красноярска, Москвы).

  1. Ошибки в управлении

1.

Употребление неверно выбранного падежа: Согласно приказа (вм. приказу), заведующий кафедры (вм. кафедрой).

  1. Ошибки в построении простого несложного предложения

1.

Личное местоимение дублирует подлежащее: Этот автор, он всегда ставит острые вопросы.

2.

Расположение слов в предложении, не соответствующее грамматическим нормам русского языка: Ученики четко должны усвоить понятия (вм. должны четко усвоить).

  1. Ошибки в построении предложения, осложненного однородными членами

1.

Рассогласование времен: Учебник дает знания, научит выделять главное. (вм. дает, учит).

2.

Объединение в одном ряду однородных членов родовых и видовых понятий: Он принес плоскогубцы, инструменты, молоток и гвозди.

3.

Парное сочетание несопоставимых понятий : Родители и друзья, коллеги и дети.

4.

Не сочетаемость опорного слова и одного из компонентов однородных членов:

Проявление заботы и помощи радовали всех (нельзя проявлять помощь).

5.

Объединение в качестве однородных полных и кратких форм прилагательных: Комната просторная и светла (вм. просторна и светла).

  1. Ошибки в построении сложносочиненного предложения

1.

Употребление противительных союзов вместо соединительных:

Тигр вскочил на спину слона, но задел лапами дрова, но дрова упали на него.

2.

Употребление лишнего союза: Онегин знал об этом, но однако ничего не предпринял.

  1. Ошибки в построении сложноподчиненного предложения

1.

Одновременное употребление подчинительных и сочинительных союзов: Как только перестал дождь, и мы отправились домой (вм. Как только перестал дождь, мы отправились домой).

2.

Пропуск указательного слова (неоправданный эллипсис): В армии Петя просился (?), где опаснее.

3.

Тавтологическое употребление союзов и союзных слов, нагромождение придаточных частей: Когда утром казаки встали, Тарас удивился, когда увидел, что среди казаков нет Андрия.

  • При вынесении грамматической ошибки эксперту необходимо делать на полях работы выпускника подробную запись, например: Г VII . 3

К 9. Классификация речевых ошибок

  1. Лексические анахронизмы (ошибки, связанные с перенесением современного слова в действительность прошлого или, наоборот, немотивированное использование архаизмов в современной речи): Уходя от войск Наполеона, помещики эвакуировали свои семьи.

  1. Вульгаризация речи ( этико-речевые ошибки):

  • Использование стилистически сниженной и нелитературной лексики (жаргонизмов, просторечий), а также инвектив (оскорблений): Политические ворюги и шестерки . Да он просто придурок.

  • Использование слов, оборотов речи и целых высказываний, в которых нашел отражение цинизм индивидуального и социального мышления: жмурик (о покойнике), пушечное мясо (о солдатах).

  • Использование слов-паразитов: Они, так сказать, решили…. Раскольников типа раскаялся… Мы как бы умные люди.

  1. Двусмысленность, неоднозначность фразы, высказывания; возможность двоякого понимания речи: Учителю надо многое сказать (Учитель должен многое сказать или учителю должны многое сказать).

  1. Канцелярские штампы (канцелярит), стилистически не мотивированное использование канцеляризмов в несвойственном им стилевом контексте: Катериной было принято решение утопиться.

  1. Нарушение лексической сочетаемости (нарушение говорящим правил соединения слов в словосочетание): одержать поражение (вм. победу), обильное общение (вм. обильный полив грядок).

  1. Ошибки в употреблении синонимов: неверный выбор одного из близких

по значению слов.

  1. Ошибки в употреблении паронимов: не различение паронимов: Вы уже ходили вешаться (вм. взвешиваться); одень пальто (вм. надень).

  1. Ошибки в употреблении антонимов: неправильный подбор антонима: не только смех, но и сострадание (вм. смех-слезы).

  1. Плеоназм ( немотивированное многословие, избыточность языковых средств для выражения данного содержания): в апреле –месяце, моя автобиография.

  1. Тавтология: неоправданное повторение в пределах одного или соседних предложений одних и тех же слов, однокоренных или созвучных слов:

Следует сказать следующее.

  1. Ошибки в выборе слов: неоправданное использование похожего слова:

А. Блок-мастер звукозаписи.

  • При вынесении речевой ошибки эксперту необходимо делать на полях работы выпускника подробную запись, например: Р2

Классификация фактических ошибок

Ф1. Искажение содержания литературного произведения, неправильное токование, неудачный подбор примеров: Счастьем для Обломова было одиночество и равнодушие. Ленский вернулся в свое имение из Англии. Базаров был нигилист и поэтому убил старуху топором.

Ф2. Неточность в цитате, отсутствие указания на автора цитаты. Неверно названный автор цитаты: Книги много значат для меня, ведь еще Ленин сказал:

«Век живи-век учись!»

Ф3. Незнание исторических и др. фактов, в том числе, временное смешение:

Великая Отечественная война 1812 года. Столица США-Нью-Йорк.

Ф4 Неточности в именах и фамилиях, прозвищах литературных героев. Искажения в названиях литературных произведений, их жанров: Тургеньев, «Тарас и Бульба», в повести Тургенева «Преступление и наказание».

  • При вынесении фактической ошибки эксперту необходимо делать на полях работы выпускника подробную запись, например: Ф2

Этические ошибки:

Речевая некорректность.

Проявление речевой агрессии:

грубое, оскорбительное высказывание; словесное выражение негативных эмоций, чувств или намерений в неприемлемой в данной речевой ситуации форме; угроза, грубое требование, обвинение, насмешка; употребление бранных слов, вульгаризмов, жаргонизмов, арго; высказывания, унижающие человеческое достоинство, выражающее высокомерие и циничное отношение к человеческой личности.

Мне хотелось бы сделать автору замечание за его неумение передавать свои мысли.

Этот текст меня бесит.

Это старье, именуемое классикой.

Логические ошибки

Л1. Сопоставление (противопоставление) двух логически неоднородных (различных по объему и по содержанию) понятий в предложении, тексте.

На уроке присутствовали директор, библиотекарь, а также Анна Петровна и Зоя Ивановна Петрова. Он облокотился спиною на батарею.

Л2.Нарушение причинно-следственных отношений.

В последние годы очень много сделано для модернизации образования, однако педагоги работают по-старому, так как вопросы модернизации решаются слабо.

Л3. Пропуск звена в объяснении, «логический скачок».

Людской поток через наш двор перекрыть вряд ли возможно. (?)А как хочется, чтобы двор был украшением школы.

Л4. Перестановка частей текста (если она не обусловлена заданием к сочинению или изложению).

Пора вернуть этому слову его истинный смысл! Честь… Но как это сделать?

Л5.

Неоправданная подмена лица, от которого ведется повествование (например, сначала от первого, затем от третьего лица).

Автор пишет о природе, описывает природу севера, вижу снега и просторы равнин.

Л6.Сопоставление логически несопоставимых понятий.

Синтаксис энциклопедических статей отличен от других научных статей.

Композиционно-текстовые ошибки

Л7.Неудачный зачин.

В этом тексте автор… (указание на предыдущий контекст с помощью указательных словоформ).

Л8. Ошибка в основной части:

  • Сближение относительно далеких мыслей в одном предложении;

  • Отсутствие последовательности в изложении; бессвязность и нарушение порядка предложений.

  • Использование разнотипных по структуре предложений, ведущее к затруднению понимания смысла.

Л9. Неудачная концовка: дублирование вывода, неоправданное повторение высказанной ранее мысли.

  • При вынесении логической ошибки эксперту необходимо делать на полях работы выпускника подробную запись, например: Л2

10

Понравилась статья? Поделить с друзьями:
  • Фиксай 100 процентов стал ошибкой
  • Фикс ошибки 268 роблокс
  • Фикс ошибки 0xc000007b
  • Фикбук ошибки прошлого
  • Фикбук ошибка природы